Anda di halaman 1dari 50

Dr

_F

aq

eh

2002 Self-Assessment Exercise


XX. Skin disorders
[Return to Category List]

Questions
Question 85.

Answer.

You are asked to evaluate a 1-day-old healthy term African-American infant who has a rash.
Findings on physical examination include scattered pustules without surrounding erythema
involving the trunk and forehead and several small hyperpigmented macules, some of which
possess a collarette of scale.
Of the following, the MOST likely diagnosis is:
A.
B.
C.
D.
E.

candidiasis
erythema toxicum
neonatal herpes simplex virus infection
Staphylococcus aureus infection
transient neonatal pustular melanosis

Question 147.

Answer.

An infant has a seizure disorder and a large, violaceous patch that involves much of the right
side of the face, including the periorbital region.
Of the following, the MOST likely diagnosis is:
A.
B.
C.
D.
E.

Kasabach-Merritt syndrome
Klippel-Trenaunay-Weber syndrome
neurofibromatosis type-1
Sturge-Weber syndrome
tuberous sclerosis

Answers
Critique 85.

Preferred Response: E
[View Question]
There are many pustular or vesicular cutaneous disorders that present in the neonatal period.
The physical findings described for the infant in the vignette are those of transient neonatal
pustular melanosis, a condition of unknown etiology that begins in utero. Pustules and small
(2 to 3 mm in diameter) hyperpigmented macules (Figure 85A) are present at birth. The
XX. Skin disorders, Self-Assessment Exercise by Category
PREP Archive on CD-ROM. Copyright 2003, American Academy of Pediatrics.

Dr

_F

aq

eh

macules are surrounded by a collarette of scale that represents the remnant of a pustule roof.
Although the clinical diagnosis usually is straightforward, if there is uncertainty, a Gram or
Wright stain of pustule fluid will reveal polymorphonuclear neutrophils (PMNs) without
organisms.
Candidiasis in the neonate may be acquired in utero or at the time of delivery. Infants
who have congenital candidiasis exhibit diffuse scaling and erythema or erythematous
papules and pustules at birth. Those who acquired infection natally have similar physical
findings that begin days to weeks after delivery. A potassium hydroxide preparation
performed on scale or a pustule roof demonstrates pseudohyphae and spores.
Erythema toxicum (Figure 85B) is a benign, self-limited eruption that occurs in
approximately 50% of term newborns. It is characterized by erythematous macules 2 to 3 cm
in diameter that have a central papule, vesicle, or pustule. The eruption typically begins at 24
to 48 hours of life and lasts for 4 to 5 days. A Wright stain (Figure 85C) of the contents of a
vesicle or pustule reveals numerous eosinophils.
Neonatal herpes simplex virus infection usually is acquired from passage through an
infected birth canal. The clinical hallmark of infection is clustered vesicles on an
erythematous base. Although lesions may be present at birth, more often they appear during
the first week of life. Viral culture is the best method for diagnosis but requires 24 hours or
more for interpretation. In experienced hands, rapid confirmation of infection can be
provided by a Tzanck smear that demonstrates multinucleated giant cells. Infection of hair
follicles by Staphylococcus aureus (eg, folliculitis) is characterized by the presence of
pustules with surrounding erythema. A Gram stain will reveal gram-positive cocci and
PMNs.
References:
Vasiloudes P, Morelli JG, Weston WL. A guide to rashes in newborns. Contemp Pediatr.
1997;14:156-167
Weston WL, Lane AT, Morelli JG. Color Textbook of Pediatric Dermatology 2nd ed. St
Louis, Mo: Mosby, Inc; 1996:330-335
Critique 147.

Preferred Response: D
[View Question]
The presence of a facial port-wine stain (PWS) and seizures is consistent with Sturge-Weber
syndrome (SWS). A PWS (Figure 147A) is a permanent vascular malformation that is
present at birth and consists of ectatic dermal blood vessels. At birth, a PWS may be pale
pink, but with time it becomes violaceous and thickened and may develop vascular nodules
that bleed. Although a PWS may occur anywhere on the body, lesions located on the face,
especially around the eye, may be associated with central nervous system (CNS) or
ophthalmic involvement. In a study of 274 patients who had PWSs involving the distribution
of the trigeminal nerve, 8% had associated CNS or eye disease manifested as seizures or
glaucoma, respectively. In all of those who had these complications, the PWS was located in
the distribution of the first (including the upper eyelid) or second (including the lower eyelid)
branches of the trigeminal nerve. Additionally, the risk for CNS or eye disease was increased
if the PWS was bilateral or, if unilateral, it involved the distribution of all three branches of
XX. Skin disorders, Self-Assessment Exercise by Category
PREP Archive on CD-ROM. Copyright 2003, American Academy of Pediatrics.

Dr

_F

aq

eh

the trigeminal nerve.


In children who have SWS, the vascular malformation involves not only the skin, but
also the ipsilateral leptomeningeal vessels, particularly those in the parieto-occipital region.
Slow flow through these vessels causes hypoxic injury that, in turn, may result in seizures or
contralateral hemiparesis. The vascular malformation also may affect the ipsilateral eye.
Abnormalities of the choroid and episcleral vessels may lead to retinal detachment or
glaucoma, respectively. For these reasons, an experienced pediatric neurologist and pediatric
ophthalmologist should be consulted whenever a child who has a facial PWS is suspected of
having SWS.
Although infants who have Kasabach-Merritt or Klippel-Trenaunay-Weber
syndromes have cutaneous vascular anomalies, clinical features of these disorders permit
their differentiation from SWS. Infants who have Kasabach-Merritt syndrome develop a
solitary, often rapidly enlarging, atypical-appearing hemangioma (actually a
hemangioepithelioma) that sequesters platelets and fibrinogen, causing thrombocytopenia
and a localized consumptive coagulopathy. Klippel-Trenaunay-Weber syndrome is a
condition in which a PWS or other vascular malformation located on an extremity is
associated with overgrowth of that extremity.
Infants who have neurofibromatosis type-1 (NF-1) or tuberous sclerosis do not exhibit
characteristic cutaneous vascular anomalies. NF-1 is an autosomal dominantly inherited
disorder; caf au lait (Figure 147B) macules are the hallmark of the disease and often are the
earliest manifestation. Children who have NF-1 also may exhibit axillary or inguinal
freckling (Figure 147C), but neurofibromas (Figure 147D) generally do not appear until
adolescence. Tuberous sclerosis, another neurocutaneous disorder, is characterized by
seizures, mental retardation, and adenoma sebaceum (Figure 147E) . The earliest cutaneous
sign of tuberous sclerosis, often present at birth, is the ash-leaf macule (Figure 147F). These
hypopigmented spots may be single or multiple and typically are located on the trunk and
extremities. Other characteristic skin findings, including the shagreen patch (Figure 147G),
adenoma sebaceum, and periungual fibromas (Figure 147H), generally do not occur until late
childhood or adolescence.
References:
Enjolras O, Mulliken JB. Vascular malformations. In: Harper J, Oranje A, Prose N, eds.
Textbook of Pediatric Dermatology. Malden, Mass: Blackwell Science, Ltd; 2000:975-1016
Orlow SJ. When to suspect a neurocutaneous disorder. Contemp Pediatr. 1995;12:59-77
Tallman B, Tan OT, Morelli JG, et al. Location of port-wine stains and the likelihood of
ophthalmic and/or central nervous system complications. Pediatrics. 1991;87:323-327

2001 Self-Assessment Exercise


XX. Skin disorders
[Return to Category List]

Questions [Print Directions](1)


XX. Skin disorders, Self-Assessment Exercise by Category
PREP Archive on CD-ROM. Copyright 2003, American Academy of Pediatrics.

Dr

_F

aq

eh

Question 1.

Answer.

A 9-month-old infant has atopic dermatitis characterized by erythematous,


chapped-appearing patches on the cheeks and extremities.
Of the following, the MOST appropriate initial treatment is
A.
B.
C.
D.
E.

administration of an oral antibiotic


administration of an oral antihistamine
administration of an oral corticosteroid
application of a low-potency corticosteroid
avoidance of cow milk protein

Question 38.

Answer.

An 8-year-old girl presents with localized hair loss of 2 months' duration. Physical
examination reveals a patch of nearly complete hair loss in the parietal scalp. The scalp is
smooth without redness, scaling, or broken hairs.
Of the following, the MOST likely explanation for these findings is
A.
B.
C.
D.
E.

alopecia areata
telogen effluvium
tinea capitis
traction alopecia
trichotillomania

Question 75.

Answer.

You are examining a 6-month-old boy who has infantile spasms. There is a family history of
tuberous sclerosis.
Of the following, the cutaneous finding MOST likely to be present in this infant is
A.
B.
C.
D.
E.

adenoma sebaceum
ash-leaf macule
forehead plaque
periungual fibroma
shagreen patch

Question 112.

Answer.

A 6-month-old infant has an erythematous, scaling eruption that involves the scalp,
retroauricular areas, axillae, and diaper region. On close inspection, you observe numerous
small (2 to 4 mm) erythematous-to-brown papules and petechiae.
Of the following, the MOST likely explanation for these findings is
A. atopic dermatitis
XX. Skin disorders, Self-Assessment Exercise by Category
PREP Archive on CD-ROM. Copyright 2003, American Academy of Pediatrics.

Dr

_F

aq

eh

B.
C.
D.
E.

Langerhans cell histiocytosis


psoriasis
seborrheic dermatitis
Wiskott-Aldrich syndrome

Question 148.

Answer.

An adolescent is concerned about areas of discolored skin that he has had for the past 2
months. Physical examination reveals numerous hypopigmented scaling macules on his chest
and back. In some areas, the macules have coalesced into hypopigmented patches.
Of the following, the MOST likely explanation for these findings is
A.
B.
C.
D.
E.

pityriasis alba
pityriasis rosea
tinea corporis
tinea versicolor
vitiligo

Question 184.

Answer.

A 7-year-old boy has been diagnosed as having neurofibromatosis type 1. He has a history of
learning disabilities. Physical examination reveals macrocephaly, five large caf au lait spots,
and kyphoscoliosis.
Of the following, the MOST likely additional finding on physical examination is
A.
B.
C.
D.
E.

"ash leaf" spot


iris Lisch nodule
neurofibroma
retinal angiomata
shagreen patch

Question 220.

Answer.

A 5-year-old boy has had a pruritic eruption for 2 weeks. His medical history is
unremarkable, and no other family members are similarly affected. Findings on physical
examination include erythematous papules, some of which are crusted, that are located
between the digits, on the volar aspects of the wrists, in the axillae, and on the penis.
Of the following, the MOST appropriate treatment is administration of
A.
B.
C.
D.
E.

cephalexin orally
corticosteroid topically
hydroxyzine orally
permethrin topically
prednisone orally

Question 257.
XX. Skin disorders, Self-Assessment Exercise by Category
PREP Archive on CD-ROM. Copyright 2003, American Academy of Pediatrics.

Answer.

Dr

_F

aq

eh

A 4-year-old girl has developed multiple lesions on her face over the past 2 months. Physical
examination reveals small (2 to 3 mm) pearly white papules. Some of the papules have a
central depression.
Of the following, the MOST appropriate management of this girl's condition is to
A.
B.
C.
D.
E.

await natural involution of the lesions


freeze the lesions using liquid nitrogen
perform curettage of each of the lesions
prescribe oral acyclovir
prescribe oral cephalexin

Answers
Critique 1.

Preferred Response: D
[View Question]
Topical steroids are employed to reduce the inflammation and itching that occur during
exacerbations of atopic dermatitis (Figure 1A), as described for the infant in the vignette.
Because their cutaneous barrier is relatively thin, infants and young children are treated
initially with a low-potency agent (eg, hydrocortisone 1%). Low- or occasionally
intermediate-strength steroids are indicated for older children, and adolescents generally
require an intermediate-strength agent. High-potency preparations rarely are required for the
treatment of atopic dermatitis in pediatric patients.
Compared with normal individuals, patients who have atopic dermatitis are colonized
more frequently and more densely with Staphylococcus aureus. When there are signs of
secondary bacterial infection, such as oozing and crusting, the oral administration of an
appropriate antibiotic (eg, a first-generation cephalosporin) is indicated. During disease
exacerbations, severe pruritus may lead to scratching that results in erosions, secondary
bacterial infection, and sleep disturbances. At such times, an oral first-generation H1
antihistamine may be administered. Although oral corticosteroids will produce clinical
improvement in patients who have atopic dermatitis, symptoms invariably return following
discontinuation of the medication. Accordingly, oral corticosteroids are virtually never
indicated because of the adverse effects associated with their chronic use and relapses when
they are discontinued.
The role of food allergy in the causation or exacerbation of atopic dermatitis remains
somewhat controversial. Recent studies indicate that as many as 40% of children referred to
pediatric dermatology or allergy clinics have significant food allergy. The foods usually
implicated are egg, peanut, milk, soy, wheat, or fish. Although avoidance of cow milk protein
or other foods is not recommended routinely for children who have atopic dermatitis, referral
for testing should be considered for those who have severe disease that fails to respond to
standard therapy. If food allergy is identified, dietary restriction may produce clinical
improvement.
References:
XX. Skin disorders, Self-Assessment Exercise by Category
PREP Archive on CD-ROM. Copyright 2003, American Academy of Pediatrics.

Dr

_F

aq

eh

Burks AW, James JM, Hiegel A, et al. Atopic dermatitis and food hypersensitivity reactions.
J Pediatr. 1998;132:132-136
Eichenfield LF, Friedlander SF. Coping with chronic dermatitis. Contemp Pediatr.
1998;15:53-80
Eigenmann PA, Sicherer SH, Borkowski TA, Cohen BA, Sampson HA. Prevalence of
IgE-mediated food allergy among children with atopic dermatitis. Pediatrics. 1998;101:e8.
Available at: http:/www.pediatrics.org/cgi/content/full/101/3/e8
Knoell KA, Greer KE. Atopic dermatitis. Pediatr Rev. 1999;20:46-52
Lapidus CS, Honig PJ. Atopic dermatitis. Pediatr Rev. 1994;15:327-332
Critique 38.

Preferred Response: A
[View Question]
Alopecia areata (Figure 38A) is characterized by one or a few patches of hair loss that
usually are located on the frontal or parietal scalp, as described for the girl in the vignette.
There is no redness, scaling, or broken hairs. Some patients lose eyebrows, eyelashes, or
other body hair, but complete loss of scalp (alopecia totalis [Figure 38B]) or body hair
(alopecia universalis) occurs rarely. As many as one third of affected children have nail
pitting (Figure 38C), shallow depressions in the nail plate. Although alopecia areata is
believed to result from immune dysregulation, it is associated only rarely with other
autoimmune disorders. Therefore, routine laboratory screening is not required. Even without
therapy, 95% of children regrow hair within 1 year, although 30% will experience a
recurrence, and the prognosis worsens with successive episodes of alopecia.
Telogen effluvium causes diffuse rather than localized alopecia. Significant stresses,
such as childbirth or febrile illness, may cause a large number of follicles to enter a resting
state. Approximately 2 to 4 months later, resting hairs begin to fall out and continue to do so
for 3 to 4 months. Lost hairs are replaced by growing hairs that cycle normally.
Tinea capitis is the most common cause of alopecia in children, typically producing
one or more patches of hair loss and scaling. In the most prevalent form, caused by
Trichophyton tonsurans, black dots , (Figure 38D) which are the remnants of infected hairs
that have broken at the scalp line, are seen within the area of alopecia. In the United States,
tinea capitis also may result from infection by Microsporum canis, an organism transmitted
from dogs or cats. In this form of disease, scaling and one or more patches of alopecia are
present with broken hairs of variable length. Because hairs do not break at the scalp line,
black dots are not observed. A potassium hydroxide preparation or fungal culture will
confirm the diagnosis of tinea capitis.
Traction is a common cause of localized hair loss among children. Tightly pulling the
hair, as might occur with braiding, may cause thinning that is noticeable in areas where the
hair is parted. Trichotillomania (Figure 38E) is the term applied to alopecia that results from
repetitive twirling or rubbing of the hair or epilation caused by plucking. The scalp is the
usual site of involvement, although the eyebrows and lashes also may be lost. Children who
have trichotillomania generally exhibit a single, irregularly shaped patch of alopecia
containing hairs of differing lengths.
References:
XX. Skin disorders, Self-Assessment Exercise by Category
PREP Archive on CD-ROM. Copyright 2003, American Academy of Pediatrics.

Dr

_F

aq

eh

Atton AV, Tunnessen WW Jr. Alopecia in children: the most common causes. Pediatr Rev.
1990;12:25-30
Hurwitz S. Disorders of hair and nails. In: Clinical Pediatric Dermatology: A Textbook of
Skin Disorders of Childhood and Adolescence. 2nd ed. Philadelphia, Pa: WB Saunders Co;
1993:484-491
Vasiloudes P, Morelli JG, Weston WL. Bald spots: remember the "big three." Contemp
Pediatr. 1997;14:77-91
Critique 75.

Preferred Response: B
[View Question]
Tuberous sclerosis (TS) is an autosomal dominant neurocutaneous disorder that may present
during the first year of life with infantile spasms. Several distinctive cutaneous lesions occur
in affected patients. The most commonly observed and the earliest lesions to appear are
hypopigmented macules (Figure 75A), which are often present at birth. Ash-leaf macules are
oval-shaped areas of reduced but not absent pigmentation that may be single or multiple and
usually are located on the trunk or extremities. Other forms of hypopigmentation include
confetti-like macules located on the pretibial surfaces and macules that appear in a segmental
or dermatomal distribution. In lightly pigmented individuals, hypopigmented macules are
recognized more easily by the use of a Wood lamp.
Adenoma sebaceum (Figure 75B) are benign tumors (angiofibromas) that begin to
appear in childhood or adolescence. They are small (one to several millimeters in diameter),
pink, dome-shaped papules that are concentrated in the nasolabial folds and on the cheeks
and chin, but they may also appear on the forehead. Individual angiofibromas may coalesce
to form elevated forehead plaques. Periungual or subungual fibromas generally appear at
puberty and may disrupt the nail bed. Fibromas (Figure 75C) also may involve the gingiva.
Shagreen patches (Figure 75D) are white or yellow, irregularly shaped, thin plaques with
accentuated skin markings that result in an orange-peel appearance. These lesions represent
hamartomas of collagenous tissue and are often located in the lumbosacral region. They may
be present at birth or may appear during childhood or adolescence.
References:
Orlow SJ. When to suspect a neurocutaneous disorder. Contemp Pediatr. 1995;12:59-79
Short MP, Adams RD. Neurocutaneous diseases. In: Fitzpatrick TB, Eisen AZ, Wolff K,
Freeberg IM, Austen KF, eds. Dermatology in General Medicine. 4th ed. New York, NY:
McGraw-Hill, Inc; 1993:2255-2259
Critique 112.

Preferred Response: B
[View Question]
Langerhans cell histiocytosis (LCH) is a disease characterized by the accumulation and
proliferation of histiocytes in various organs. The term LCH encompasses conditions
previously referred to as histiocytosis X, eosinophilic granuloma of bone,
Hand-Schller-Christian disease, and Letterer-Siwe disease. LCH exhibits great clinical
heterogeneity; histiocytic proliferation may be limited to skin or bone (Figure 112A) or affect
XX. Skin disorders, Self-Assessment Exercise by Category
PREP Archive on CD-ROM. Copyright 2003, American Academy of Pediatrics.

Dr

_F

aq

eh

multiple organ systems. Some forms resolve spontaneously; others may be lethal. The skin
(Figure 112B) is the initial site of involvement in up to 50% of affected children and is the
only system affected in a minority of patients. The scalp, flexural areas, and diaper region
exhibit erythematous, scaling patches as well as discrete brown papules, petechiae, and
erosions. The infant described in the vignette has typical cutaneous lesions of LCH.
Individual lesions tend to heal with scarring or atrophy. A cutaneous biopsy is necessary to
confirm the diagnosis.
Infants who have atopic dermatitis exhibit erythematous, minimally scaling patches
on the trunk (Figure 112C), extremities (Figure 112D), face, and occasionally the scalp. In
contrast to LCH, brown papules and petechiae are not seen, and flexural areas and the diaper
region tend to be spared. Clinical features also allow differentiation of LCH from psoriasis.
Characteristic psoriatic lesions are erythematous, scaling plaques and papules that do not
cause scarring or atrophy. Although the distribution of seborrheic lesions is similar to that of
LCH, seborrheic dermatitis (Figure 112E) produces confluent salmon-pink scaling patches,
not discrete papules or petechiae. Wiskott-Aldrich syndrome is an X-linked disorder
characterized by frequent infections, bleeding caused by thrombocytopenia, and atopic
dermatitis. The associated abnormalities and the cutaneous findings, which are more
consistent with atopic dermatitis, differentiate it from LCH.
References:
Arico M, Egeler RM. Clinical aspects of Langerhans cell histiocytosis. Hematol Oncol Clin
North Am. 1998;12:247-258
Munn S, Chu AC. Langerhans cell histiocytosis of the skin. Hematol Oncol Clin North Am.
1998;12:269-286
Weston WL, Lane AT, Morelli JG. Color Textbook of Pediatric Dermatology. 2nd ed. St
Louis, Mo: Mosby-Year Book, Inc; 1996:200-203
Critique 148.

Preferred Response: D
[View Question]
The presence of numerous hypopigmented scaling macules (Figure 148A) on the chest and
back of an adolescent, as described for the patient in the vignette, suggests a diagnosis of
tinea versicolor. This common superficial fungal infection is caused by Pityrosporum
orbiculare, also known as Malassezia furfur. In its round or spore form, P orbiculare is a
common inhabitant of the normal skin in 90% to 100% of asymptomatic adults. For reasons
that are unclear, the organism forms large numbers of hyphae in some individuals, resulting
in clinical disease.
Tinea versicolor occurs most commonly in adolescents and young adults and typically
involves the chest, back, neck, upper arms, and occasionally the groin. Lesions may be hypoor hyperpigmented and usually possess a fine, powdery scale. The characteristic appearance
and distribution of lesions generally permit a clinical diagnosis, but it can be confirmed by a
potassium hydroxide preparation that reveals spores and short hyphae, so-called spaghetti
and meat balls (Figure 148B). The diagnosis also may be confirmed by Wood lamp
examination that reveals a yellow or gold fluorescence.
Several disorders may mimic tinea versicolor. Pityriasis alba is the term applied to the
XX. Skin disorders, Self-Assessment Exercise by Category
PREP Archive on CD-ROM. Copyright 2003, American Academy of Pediatrics.

Dr

_F

aq

eh

hypopigmented macules that may be observed on the faces of individuals who have atopic
dermatitis. These lesions represent areas of postinflammatory hypopigmentation and, unlike
those of tinea versicolor, have indistinct margins. Pityriasis rosea (Figure 148C) is
characterized by numerous erythematous, oval, thin plaques arranged with long axes parallel
to skin lines. On the back, the arrangement of lesions may be reminiscent of the branches of a
fir tree. Patients who have tinea corporis (Figure 148D) exhibit one or a few scaling annules,
often with central clearing, whose borders are elevated and erythematous. In contrast to tinea
versicolor, vitiligo is characterized by well-defined macules or patches that are completely
depigmented and lack scale. Lesions tend to involve periorificial areas (eg, around the eyes
and mouth) as well as the hands and feet.
References:
Rasmussen JE. Cutaneous fungus infections in children. Pediatr Rev. 1992;13:152-156
Sunenshine PJ, Schwartz RA, Janniger CK. Tinea versicolor: an update. Cutis.
1998;61:65-68, 71-72
Weston WL, Lane AT, Morelli JG. Color Textbook of Pediatric Dermatology. 2nd ed. St
Louis, Mo: Mosby-Year Book, Inc; 1996:79-80
Critique 184.

Preferred Response: B
[View Question]
Neurofibromatosis type 1 (NF-1) is an autosomal dominant disorder that can present in
childhood with caf au lait spots and learning disabilities or developmental delay. Another
manifestation of the disorder is iris Lisch nodules, which are melanocytic hamartomas that
are pathognomonic for NF-1. Other findings include axillary freckling, which usually occurs
later in childhood, and cutaneous neurofibromas that appear around the time of puberty.
Caf au lait spots are present in more than 99% of affected patients and frequently are
the initial manifestations of the disorder. They increase in size and frequency with age, and
the finding of six or more caf au lait spots greater than 0.5 to 1.0 cm in diameter is one of
the diagnostic criteria for NF-1. Mental deficiency, with an intelligence quotient below 70,
occurs in about 10% of patients, and learning disabilities are detected in another 25%.
Macrocephaly may occur and may be due to hydrocephalus. Therefore, patients in whom the
head circumference is increasing should have imaging studies to determine the etiology.
NF-1 is characterized by marked heterogeneity among affected individuals, even
within the same family. Therefore, careful examination of all at-risk relatives is warranted to
identify others who are affected. In particular, the parents of an affected child must be
examined carefully to provide them with information about recurrence risks in future
offspring. It has been estimated that 25% to 50% of patients who have NF-1 have a new
mutation.
Ash leaf spots, retinal angiomata, and shagreen patches are manifestations of
tuberous sclerosis. The ash leaf spot is a hypopigmented macule that is one of the earliest
skin lesions detected in affected patients, and shagreen patches are raised, leathery patches in
the lumbosacral area.
References:
XX. Skin disorders, Self-Assessment Exercise by Category
PREP Archive on CD-ROM. Copyright 2003, American Academy of Pediatrics.

Dr

_F

aq

eh

DeBella K, Szudek J, Friedman JM. Use of the National Institutes of Health Criteria for
Diagnosis of Neurofibromatosis 1 in Children. Pediatrics. 2000;105:608-614
Jones KL. Neurofibromatosis syndrome. In: Smith's Recognizable Patterns of Human
Malformation. 5th ed. Philadelphia, Pa: WB Saunders Co; 1997:508-509
Ragge NK. Clinical and genetic patterns of neurofibromatosis 1 and 2. Br J Ophthalmol.
1993;77:662-672
Critique 220.

Preferred Response: D
[View Question]
Scabies is a pruritic eruption cause by infestation with the mite (Figure 220A) Sarcoptes
scabiei. It usually is acquired through close contact with an infected individual, although
transmission by fomites such as clothing or bedding is possible. Children, adolescents, and
adults exhibit erythematous papules, nodules, or burrows (Figure 220B) in the interdigital
webs (Figure 220C), the flexural areas of the wrists (Figure 220D) and elbows, the axillae, at
the belt line, beneath the breasts in women, and on the penis (Figure 220E) in males. In
infants, erythematous papules, vesicles, or pustules are distributed in a generalized fashion on
the trunk (Figure 220F), extremities (Figure 220G), and head.
The treatment of choice, particularly for infants and toddlers, is permethrin cream 5%.
It is applied from the neck down; in infants, the head and face also are treated. Although a
single application left on the skin for 8 to 14 hours is approximately 90% effective, most
clinicians advise a second treatment 7 days later for symptomatic individuals. When scabies
is diagnosed, all close contacts and family members should be treated simultaneously, even if
they are asymptomatic. The absence of symptoms does not preclude the possibility of
infestation because 10 to 30 days are required to become sensitized to the mite. Failing to
treat all family members may result in cycles of reinfestation. Following treatment, all
clothing and bedding should be washed in hot water. Clothing that cannot be laundered may
be dry cleaned or stored in a plastic bag for 2 weeks. Families should be counseled that
itching may persist for several weeks after effective treatment.
Cephalexin is indicated only when the lesions have become secondarily infected.
Adjunctive therapeutic measures for scabies include the application of topical corticosteroids
for eczematized lesions and the use of an oral antihistamine such as hydroxyzine to provide
sedation and relief from pruritus. Oral steroids are not indicated for the treatment of scabies.
References:
Peterson CM, Eichenfield LF. Scabies. Pediatr Ann. 1996;25:97-100
Rasmussen JE. Scabies. Pediatr Rev. 1994;15:110-114
Critique 257.

Preferred Response: A
[View Question]
Molluscum contagiosum is a common cutaneous infection caused by a pox virus. Infection is
thought to result from direct contact, although the role of fomites cannot be excluded. The
incubation period is 2 to 7 weeks, and lesions may persist for as long as 12 to 18 months
before resolving spontaneously. Lesions of molluscum contagiosum are translucent or white
XX. Skin disorders, Self-Assessment Exercise by Category
PREP Archive on CD-ROM. Copyright 2003, American Academy of Pediatrics.

Dr

_F

aq

eh

papules (Figure 257A) that, at first glance, may resemble vesicles or pustules. Generally they
measure from one to a few millimeters in diameter and often have a central dimple or
umbilication, although this finding may be absent when lesions are very small. Frequently
there is an area of erythema and mild scaling surrounding the lesions. This may represent a
primary molluscum dermatitis or the appearance of lesions in an area of active atopic
dermatitis, which is common.
There are a variety of treatment options for molluscum contagiosum. Management
should be individualized and based on several factors, including how troubling the lesions are
for the patient and parents, the duration of lesions, whether the lesions are increasing in
number, and the patients ability to tolerate painful procedures. Because the infection
eventually resolves spontaneously, no intervention is a reasonable option for an otherwise
healthy immunocompetent individual, such as the girl described in the vignette. Parents
should realize, however, that not treating lesions may permit the spread of infection to others,
although the degree of contagion is not known. In addition, the number of lesions may
increase, particularly if there is underlying atopic dermatitis.
If a decision to treat is made, parents should be advised that available therapies may
eradicate existing lesions but not the underlying infection. Therefore, new lesions may
continue to appear for several months. Freezing or curettage of the lesions is effective, but
the associated discomfort is tolerated poorly by most young children. Prior application of a
eutectic mixture of lidocaine and prilocaine may reduce the discomfort of these procedures.
Acyclovir is not indicated for the treatment of molluscum contagiosum, and cephalexin
should be prescribed only if there are signs of secondary bacterial infection.
References:
Krowchuk DP. Warts and molluscum contagiosum. In: Burg FD, Wald ER, Ingelfinger JR,
Polin RA, eds. Gellis & Kagan's Current Pediatric Therapy. 16th ed. Philadelphia, Pa: WB
Saunders Co; 1999:970-974
Prasad SM. Molluscum contagiosum. Pediatr Rev. 1996;17:118-119
Weston WL, Lane AT, Morelli JG. Color Textbook of Pediatric Dermatology. 2nd ed. St
Louis, Mo: Mosby-Year Book, Inc; 1996:126-127

2000 Self-Assessment Exercise


XX. Skin disorders
[Return to Category List]

Questions [Print Directions](2)


Question 25.

Answer.

A 6-month-old infant has had a rash for the past 10 days. His mother reports that he has been
somewhat fussier than usual, but he has had no fever or other symptoms. No other family
members are similarly affected. Findings on physical examination include numerous
XX. Skin disorders, Self-Assessment Exercise by Category
PREP Archive on CD-ROM. Copyright 2003, American Academy of Pediatrics.

Dr

_F

aq

eh

erythematous papules on the trunk and extremities and both erythematous papules and
pustules on the palms and soles.
Of the following, the MOST appropriate treatment is topical
A.
B.
C.
D.
E.

crotamiton for the infant only


lindane 1% for the infant only
lindane 1% for the infant and all close contacts
permethrin 5% for the infant only
permethrin 5% for the infant and all close contacts

Question 50.

Answer.

A 14-year-old boy who has tuberous sclerosis comes to you for evaluation of a rash on the
face. The only medication he is taking is felbamate for management of seizures. Examination
of his face reveals numerous small (1 to 3 mm), pink papules in the malar areas bilaterally.
Of the following, the MOST likely explanation for these findings is
A.
B.
C.
D.
E.

acne vulgaris
adenoma sebaceum
drug-induced acne
flat warts
molluscum contagiosum

Question 71.

Answer.

An 8-year-old boy presents with localized loss of scalp hair of 2 months' duration. Physical
examination reveals a patch of alopecia within which hairs are of differing lengths. The
remainder of the scalp is normal, with no evidence of erythema, scaling, or "black dot" hairs.
Of the following, the MOST likely explanation for these findings is
A.
B.
C.
D.
E.

alopecia areata
telogen effluvium
tinea capitis
traction alopecia
trichotillomania

Question 90.

Answer.

A 10-month-old African-American infant has a 4-month history of a pruritic, relapsing skin


eruption. Physical examination reveals slightly erythematous, chapped-appearing patches on
the cheeks and hypopigmented patches on the trunk. Gray, slightly scaling patches appear on
extensor surfaces of the extremities. The scalp and diaper area are spared.
Of the following, the MOST likely explanation for these findings is
A. atopic dermatitis
B. contact dermatitis
XX. Skin disorders, Self-Assessment Exercise by Category
PREP Archive on CD-ROM. Copyright 2003, American Academy of Pediatrics.

Dr

_F

aq

eh

C. psoriasis
D. scabies
E. seborrheic dermatitis
Question 113.

Answer.

An infant has a port-wine stain on the face. His parents are concerned about the appearance
of the lesion and request advice about available treatments.
Of the following, the modality that offers the BEST cosmetic palliation is
A.
B.
C.
D.
E.

cryosurgery
excision and grafting
pulsed dye laser
radiation therapy
systemic corticosteroid therapy

Question 131.

Answer.

A 15-year-old boy presents for treatment of a pruritic eruption of 4 days' duration. He


recently returned from a camping trip during which he wore a short-sleeved shirt and short
pants. On physical examination, you find numerous erythematous papules and vesicles, some
in a linear array, on the arms and legs.
Of the following, the MOST appropriate treatment is
A.
B.
C.
D.
E.

cephalexin orally for 7 days


diphenhydramine topically for 5 days
hydrocortisone 1% topically for 5 days
mupirocin topically for 7 days
prednisone orally for 12 days

Question 153.

Answer.

You are asked to evaluate a 6-year-old boy who has dry skin. Past history is remarkable for
mild atopic dermatitis. The boy's father also has scaly skin and allergic rhinitis. Physical
examination reveals generally dry skin and thin scales that have a "pasted-on" appearance on
the extensor surfaces of the legs and buttocks.
Of the following, the MOST likely explanation for these findings is
A.
B.
C.
D.
E.

atopic dermatitis
bullous ichthyosis
ichthyosis vulgaris
lamellar ichthyosis
psoriasis

Question 176.
XX. Skin disorders, Self-Assessment Exercise by Category
PREP Archive on CD-ROM. Copyright 2003, American Academy of Pediatrics.

Answer.

Dr

_F

aq

eh

A 5-year-old African-American boy is brought to you for evaluation of a mass on the scalp
and hair loss. Physical examination reveals a tender, boggy mass on the scalp with overlying
alopecia and enlarged posterior cervical lymph nodes.
Of the following, the MOST appropriate therapy is
A.
B.
C.
D.
E.

cephalexin orally
griseofulvin orally
incision and drainage
mupirocin topically
oxacillin parenterally

Question 195.

Answer.

A 4-year-old girl has a 5-day history of yellow, crusted erosions beneath the nares and on the
cheek, chin, and upper extremities. The child is otherwise well and has no known allergies to
medications.
Of the following, the MOST appropriate treatment is
A.
B.
C.
D.
E.

acyclovir topically
cephalexin orally
hydrocortisone 1% topically
mupirocin topically
penicillin orally

Question 218.

Answer.

Five days after the onset of an upper respiratory tract infection, an 8-month-old infant
developed irritability, fever, and erythema of the skin that involved the face, chest, and upper
extremities. There is no history of trauma, and the infant is receiving no medications. On
examination, vital signs are normal except for a temperature of 39.6C (103.3F). The skin is
tender to the touch, and there is crusting about the mouth. Rubbing the skin causes epidermal
separation.
Of the following, the MOST likely diagnosis is
A.
B.
C.
D.
E.

scald burn
staphylococcal scalded skin syndrome
streptococcal scarlet fever
toxic epidermal necrolysis
toxic shock syndrome

Question 237.

Answer.

A 6-month-old infant has a 2-month history of an eruption characterized by erythematous


erosions located around the mouth, on the hands and feet, and in the perineum. There is
crusting of the scalp, and hair is sparse. The infant has experienced diarrhea and poor weight
gain for the past month. Past medical history is unremarkable. The infant was weaned from
XX. Skin disorders, Self-Assessment Exercise by Category
PREP Archive on CD-ROM. Copyright 2003, American Academy of Pediatrics.

Dr

_F

aq

eh

human milk to formula at the age of 2 months.


Of the following, the MOST likely explanation for these findings is
A.
B.
C.
D.
E.

acrodermatitis enteropathica
atopic dermatitis
impetigo
irritant dermatitis
seborrheic dermatitis

Question 260.

Answer.

A 15-year-old girl requests treatment for acne. On physical examination you note mild
inflammatory and comedonal acne on the face. There are no scars or cysts, and the chest and
back are free of lesions.
Of the following, the preferred INITIAL treatment is
A.
B.
C.
D.
E.

benzoyl peroxide gel 5% topically


clindamycin lotion 1% topically
salicylic acid wash 2% topically
tetracycline orally
tretinoin cream 0.025% topically

Answers
Critique 25

Preferred Response: E
[View Question]
Scabies is a pruritic eruption caused by infestation with the mite Sarcoptes scabiei. It usually
is acquired through close contact with an infected individual, although transmission by
fomites such as clothing or bedding is possible. In infants, scabies is characterized by
erythematous papules, vesicles, or pustules that are distributed in a generalized fashion on the
trunk, extremities, and head, as described for the infant in the vignette. Because they are
damaged by scratching and secondary eczematization, burrows may not be seen. Older
children, adolescents, and adults exhibit erythematous papules, nodules, or burrows in the
interdigital webs, the flexural areas of the wrists and elbows, the axillae, at the belt line,
beneath the breasts in women, and on the penis in males.
The treatment of choice, particularly for infants and toddlers, is permethrin cream 5%.
The medication should be applied to the entire body surface of infants, including the head
and face and under the nails for 8 to 14 hours. Toddlers and older patients should have the
medication applied from the neck down. Although a single treatment is approximately 90%
effective, most clinicians advise a second treatment 7 days following initial application for
patients who were symptomatic.
Lindane 1% also is effective in eradicating scabies, although concerns about potential
neurotoxicty, usually caused by excessive application, preclude its use in infants or young
children. Like permethrin, lindane is applied from the neck down and washed off in 8 to 14
XX. Skin disorders, Self-Assessment Exercise by Category
PREP Archive on CD-ROM. Copyright 2003, American Academy of Pediatrics.

Dr

_F

aq

eh

hours; the treatment should be repeated in 1 week for patients who were symptomatic. The
safety of using either permethrin or lindane in pregnant women has not been established.
Crotamiton is much less effective than either permethrin or lindane. If prescribed, many
clinicians employ a 5 day treatment course rather than the 2 day regimen suggested by the
manufacturer.
When scabies is diagnosed, all close contacts and family members should be treated
simultaneously, even if they are asymptomatic. The absence of symptoms does not preclude
the possibility of infestation because 10 to 30 days are required to become sensitized to the
mite. Failing to treat all family members may result in cycles of reinfestation. Accordingly,
the infant described in the vignette and all close contacts should be treated with permethrin
5%.
Following treatment, all clothing and bedding should be washed in hot water.
Clothing that cannot be laundered may be dry cleaned or stored in a plastic bag for 2 weeks.
Families should be counseled that itching may persist for several weeks after effective
treatment. If the itching is not accompanied by the appearance of new lesions, this represents
an ongoing allergic response to the mite rather than treatment failure. If itching poses a
problem, patients may be treated with an oral antihistamine or the application of an
appropriate topical corticosteroid.
References:
Peterson CM, Eichenfield LF. Scabies. Pediatr Ann. 1996;25:97-100
Rasmussen JE. Scabies. Pediatr Rev. 1994;15:110-114
Critique 50

Preferred Response: B
[View Question]
Pink papules located in the malar area of a patient who has tuberous sclerosis suggest a
diagnosis of adenoma sebaceum. These lesions actually are angiofibromas, benign tumors
that begin to appear during childhood or adolescence but occasionally are observed at birth.
Individual lesions are small (1 to several millimeters in diameter), pink, dome-shaped papules
that are concentrated in the nasolabial folds and on the cheeks and chin, although they may
appear on the forehead.
Adenoma sebaceum may be confused with the inflammatory lesions of acne vulgaris,
and in an adolescent, the conditions may coexist. However, unlike acne lesions, adenoma
sebaceum do not involute and may coalesce to form plaques. In addition, individuals who
have acne generally exhibit a variety of lesions, including inflammatory pustules and papules
as well as open (blackheads) and closed (whiteheads) comedones. Finally, unlike
adenoma sebaceum, the lesions of acne are distributed widely on the face and may be present
on the chest and back.
An acne-like eruption induced by systemic corticosteroids may mimic adenoma
sebaceum. However, steroid acne is linked temporally to drug administration, and lesions
usually are pustules in the same stage of development that are distributed widely over the
face and trunk. A similar eruption may occur during therapy with lithium carbonate,
anticonvulsants (eg, diphenylhydantoin, phenobarbital, and trimethadione), and isoniazid.
Felbamate has not been associated with drug-induced acne.
XX. Skin disorders, Self-Assessment Exercise by Category
PREP Archive on CD-ROM. Copyright 2003, American Academy of Pediatrics.

Dr

_F

aq

eh

Viral infections such as flat warts and molluscum contagiosum often involve the face.
Flat warts are multiple, small (measuring a few millimeters in diameter), smooth,
flesh-colored or pink, thin plaques that often are located on the face and extremities.
Molluscum contagiosum is characterized by one or more translucent or white papules that
initially may resemble vesicles or pustules. Careful inspection and palpation, however, reveal
the lesions to be solid. Individual papules may grow to 1 cm or greater in diameter, but
typically measure from 1 to a few millimeters. A central dimple or umbilication is the
hallmark of molluscum lesions, although it may be absent in very small papules.
References:
Hurwitz S. Tuberous sclerosis. In: Clinical Pediatric Dermatology: A Textbook of Skin
Disorders of Childhood and Adolescence. 2nd ed. Philadelphia, Pa: WB Saunders Co;
1993:629-633
Orlow SJ. When to suspect a neurocutaneous disorder. Contemporary Pediatrics.
1995;12:59-77
Critique 71

Preferred Response: E
[View Question]
Trichotillomania is the term applied to alopecia that results from repetitive twirling or
rubbing of the hair or epilation caused by plucking. The scalp is the usual site of
involvement, although the eyebrows and lashes also may be lost. Most often, the condition is
the result of a habit or a response to stress and is not a sign of significant psychopathology, as
once believed. Children who have trichotillomania generally exhibit a single, irregularly
shaped patch of alopecia. Examination of the scalp reveals an area of alopecia without
erythema or scaling containing hairs of differing lengths, as described for the boy in the
vignette.
A number of conditions may produce hair loss and, therefore, can be confused with
trichotillomania. Tinea capitis is the most common cause of alopecia in children, typically
producing one or more patches of hair loss and scaling. In the most prevalent form caused by
Trichophyton tonsurans, black dots, the remnants of infected hairs that have broken at the
scalp line, are seen within the area of alopecia. In the United States, tinea capitis also may
result from infection by Microsporum canis, an organism transmitted from dogs or cats. In
this form of disease, scaling and alopecia are present, but hairs break further from the scalp
and, therefore, black dot hairs are not observed. A potassium hydroxide preparation or
fungal culture can be used to confirm the diagnosis of tinea capitis.
Another common cause of localized hair loss among children is traction. Tightly
pulling the hair, as might occur with braiding, may cause thinning that is noticeable in the
areas where hair is parted. Hair loss also may result from alopecia areata, a disorder that is
presumed to be autoimmune. Most children exhibit one or more areas of complete hair loss
without scaling or inflammation of the scalp. In rare cases, there may be extensive or
complete loss of scalp hair (alopecia totalis) and other body hair (alopecia universalis).
Pitting of the nails that appears as shallow depressions is present in 50% of children who
have alopecia areata.
Telogen effluvium causes diffuse rather than localized alopecia. Ordinarily, nearly
XX. Skin disorders, Self-Assessment Exercise by Category
PREP Archive on CD-ROM. Copyright 2003, American Academy of Pediatrics.

Dr

_F

aq

eh

90% of scalp hair is in a growing or anagen phase, with the remainder in a resting or telogen
phase. Each scalp hair grows for approximately 3 years after which the follicle enters a
resting state for a period of 2 to 3 months. However, with a significant stress, such as birth or
an illness associated with high fever, a large number of follicles are converted to the resting
state. Approximately 2 to 4 months after the stressful event, the resting hairs begin to fall out
and continue to do so for 3 to 4 months. Lost hairs are replaced by growing hairs that cycle in
a normal fashion. Although findings on the history and physical examination generally are
sufficient to suggest the presence of telogen effluvium, the diagnosis may be confirmed by
microscopic examination of the hair. A number of hairs are pulled from the scalp and placed
in oil or a mounting solution on a glass slide. In telogen effluvium, there will be a larger
proportion of resting hairs. These are characterized by loss of the normal root pigment within
the core of the hair, a hair root that has a smaller caliber than the remainder of the hair shaft,
and loss of the external root sheath.
References:
Atton AV, Tunnessen WW Jr. Alopecia in children: the most common causes. Pediatr Rev.
1990;12:25-30
Hurwitz S. Disorders of hair and nails. In: Clinical Pediatric Dermatology: A Textbook of
Skin Disorders of Childhood and Adolescence. 2nd ed. Philadelphia, Pa: WB Saunders Co;
1993:484-491
Stein DH. Tineas-superficial dermatophyte infections. Pediatr Rev. 1998;19:368-372
Vasiloudes P, Morelli JG, Weston WL. Bald spots: remember the big three. Contemporary
Pediatrics. 1997; 14:76-91
Critique 90

Preferred Response: A
[View Question]
Atopic dermatitis is one of the most common pediatric skin disorders. It generally begins
during infancy or childhood; 90% of affected patients present before 5 years of age. The
diagnosis is made clinically, based on the presence of three or more of the following: typical
morphology and distribution of lesions, pruritus, chronic relapsing course, and a family or
personal history of atopic disorders.
The hallmark of atopic dermatitis is itching that can be severe and results in erosions,
thickening of the skin (lichenification), changes in skin color, and secondary bacterial
infection. The distribution of lesions varies with age. Infants and toddlers, for example, often
have involvement of the face, trunk, and extremities and sparing of the diaper area, as
described for the infant in the vignette. During childhood, lesions are concentrated in flexural
areas, such as the antecubital and popliteal fossae, wrists, and ankles. Adolescents continue to
exhibit flexural involvement, but often develop lesions on the hands, face, and neck.
The patient's racial background also influences the appearance of lesions. In
Caucasians, lesions are erythematous, somewhat scaly or crusted papules, patches, or thin
plaques. However, in more deeply pigmented individuals, erythema is less obvious, and
lesions may appear gray. In addition, persons of color tend to have a more papular eruption as
well as postinflammatory hypo- or hyperpigmentation that frequently requires months to
resolve.
XX. Skin disorders, Self-Assessment Exercise by Category
PREP Archive on CD-ROM. Copyright 2003, American Academy of Pediatrics.

Dr

_F

aq

eh

These clinical features generally allow differentiation of atopic dermatitis from other
disorders. Although contact dermatitis may produce a pruritic eruption, a history of exposure
to an offending agent, the presence of vesicles and bullae in acute forms of disease, and the
location of lesions in a pattern compatible with application of an allergen permit an accurate
diagnosis.
In contrast to atopic dermatitis, the lesions of psoriasis are well-defined plaques that
have thick, adherent scale. Furthermore, scaling of the scalp is common in patients who have
psoriasis, and the diaper area often is involved, although scale may be absent in this region.
Scabies may be mistaken for atopic dermatitis, particularly during infancy, because
both conditions may have a generalized distribution. However, the lesions of scabies are
individual, erythematous papules and, occasionally, vesicles or pustules. A history of similar
lesions in other family members favors a diagnosis of scabies, but this is not always present.
If there is concern about scabies, examining a mineral oil preparation for mites, eggs, or fecal
material can be valuable.
Seborrheic dermatitis may mimic atopic dermatitis during infancy. However, infants
who have seborrheic dermatitis often exhibit significant scaling of the scalp, involvement of
flexural areas, and lesions covered with a greasy scale. The natural history of the disorders
also differs. Atopic dermatitis usually persists into childhood; seborrheic dermatitis resolves
by the end of the first year of life.
References:
Friedlander SF. Contact dermatitis. Pediatr Rev. 1998;19: 166-171
Halbert AR. The practical management of atopic dermatitis in children. Pediatr Ann.
1996;25:72-80
Rabinowitz LG, Esterly NB. Atopic dermatitis and ichthyosis vulgaris. Pediatr Rev.
1994;15:220-226
Critique 113

Preferred Response: C
[View Question]
Port-wine stains (PWS) are permanent vascular malformations that are present at birth in
0.3% of infants. Lesions may occur anywhere on the body, but most often they are located on
the face. Over time, PWS on the face darken, often develop vascular nodules that bleed, and
may cause overgrowth of tissue. Several studies indicate, not surprisingly, that facial PWS
frequently are a source of emotional distress.
Most experts agree that the treatment of choice for a patient who has significant facial
PWS is therapy with a pulsed dye laser. The majority of children experience a reduction in
the size of the PWS following treatment and a few will have complete resolution. Factors that
affect the treatment outcome include the location and size of the lesion and the age at which
therapy was begun. Treatment-related reductions in PWS size appear to be greatest when a
lesion is located on the central forehead, followed in order by those on the peripheral face
(eg, lateral forehead, cheeks, temples, jaws and chin), central face (eg, nose, upper lip, and
fatty cheeks), and those that are mixed in location. Smaller lesions are more likely to
diminish in size and become lighter after treatment than larger lesions. In one study, children
XX. Skin disorders, Self-Assessment Exercise by Category
PREP Archive on CD-ROM. Copyright 2003, American Academy of Pediatrics.

Dr

_F

aq

eh

who had lesions smaller than 20 cm2 had a mean reduction in PWS size of 60%, and 32%
achieved complete resolution. In contrast, children who had larger PWS had a mean size
reduction of only 41%, and none had complete resolution. Finally, early treatment appears to
be associated with the best outcome. When laser treatment was initiated during the first year
of life, one group of investigators documented a mean PWS size reduction of 65% and
complete resolution in 32% of treated infants. In contrast, those who began treatment
between 2 and 6 years of age had a mean PWS size reduction of 54%, and only 17% achieved
complete clearance.
Other treatment modalities, such as cryosurgery, radiation therapy, or excision and
grafting, are not indicated for the child described in the vignette. Although systemic
corticosteroids may be effective in the treatment of infants who have hemangiomas that
threaten vital structures, they are of no value in the management of PWS.
References:
Morelli JG, Weston WL, Huff JC, Yohn JJ. Initial lesion size as a predictive factor in
determining the response of port-wine stains in children treated with the pulsed dye laser.
Arch Pediatr Adolesc Med. 1995;149:1142-1144
Nguyen CM, Yohn JJ, Huff C, Weston WL, Morelli JG. Facial port wine stains in childhood:
prediction of the rate of improvement as a function of the age of the patient, size and location
of the port wine stain and the number of treatments with the pulsed dye (585 nm) laser. Br J
Dermatol. 1998;138:821-825
Strauss RP, Resnick SD. Pulsed dye laser therapy for port-wine stains in children:
psychosocial and ethical issues. J Pediatr. 1993;122:505-510
Critique 131

Preferred Response: E
[View Question]
The appearance of vesicles, bullae, and erythematous papules in a linear arrangement on
exposed areas, as described for the adolescent in the vignette, suggests contact dermatitis due
to plant allergens (eg, urushiol present in poison ivy, poison oak, or poison sumac). Contact
dermatitis also may result from exposure to agents such as nickel (present in jewelry, belt
buckles, or clothing snaps); potassium dichromate (present in some shoes); neomycin,
thimerosal, or formaldehyde (used in topical medications); or Balsam of Peru or other
fragrances (used in perfumes or soaps).
Contact dermatitis occurs when an antigen penetrates the epidermis and sensitizes T
lymphocytes, which then circulate through the skin and other organ systems. For potent
antigens such as urushiol, sensitization takes only 7 to 10 days; weaker agents may require
multiple exposures over many weeks. If sensitization occurs, re-exposure to the antigen will
cause contact dermatitis to develop within 12 to 24 hours.
Potent antigens such as urushiol typically cause an acute dermatitis that consists of
vesicles, bullae, erythematous papules, and edema. New lesions may continue to appear over
several days. If untreated, the dermatitis may persist for 3 to 4 weeks. In contrast, weaker
antigens produce a subacute dermatitis that is characterized by erythema, scaling, and
lichenification (ie, thickening of the skin caused by scratching).
XX. Skin disorders, Self-Assessment Exercise by Category
PREP Archive on CD-ROM. Copyright 2003, American Academy of Pediatrics.

Dr

_F

aq

eh

The key to recognizing contact dermatitis is the observation that the eruption is
limited to certain skin areas. Linear vesicles or bullae on exposed surfaces suggest exposure
to plant allergens. Nickel dermatitis occurs at sites of contact with jewelry (eg, on the
earlobes or other sites of piercing, neck, or wrists) or below the umbilicus where there is
contact with a belt buckle or clothing snap. An eruption on the dorsa of the feet should raise
the suspicion of a shoe dermatitis.
If the contact dermatitis is mild, a mid-potency topical corticosteroid (eg,
triamcinolone acetonide 0.1% or fluocinolone acetonide 0.01%) may be applied twice daily;
low-potency preparations (eg, hydrocortisone 1%) are ineffective. However, when more than
10% to 15% of the body surface is involved, as described for the adolescent in the vignette,
oral prednisone may be indicated. An initial dose of 1 mg/kg should be administered once
daily, then tapered by 20% to 25% every 3 days to complete a 12 to 21 day course.
Discontinuing therapy earlier may result in a flare of symptoms. Other treatment modalities
also may be beneficial. When vesicles rupture, drying can be promoted by taking tepid baths
or applying cool compresses or shake lotions (eg, calamine). Oral antihistamines produce a
sedative effect that offers relief from itching. Topical anesthetics containing benzocaine and
topical antihistamines (eg, diphenhydramine) should be avoided because they may induce a
contact dermatitis that could complicate the primary process. Unless secondary bacterial
infection is present, oral (eg, cephalexin) or topical (eg, mupirocin) antibiotics are not
indicated.
Prevention of further episodes is an important part of the management of patients who
have a contact dermatitis. Individuals who have experienced plant dermatitis should learn to
recognize and avoid poison ivy, sumac, and oak. When exposure may be unavoidable (eg,
during hikes or camping trips), wearing protective clothing or applying a barrier preparation,
such as Ivy Block or Stokogard, may be useful. These products prevent the resin from
reaching the skin and are applied to areas likely to be exposed prior to potential contact.
References:
Friedlander SF. Contact dermatitis. Pediatr Rev. 1998;19: 166-171
Hogan PA, Weston WL. Allergic contact dermatitis in children. Pediatr Rev.
1993;14:240-243
Williford PM, Sheretz EF. Poison ivy dermatitis. Nuances in treatment. Arch Fam Med.
1994;3:184-188
Critique 153

Preferred Response: C
[View Question]
Ichthyosis is a term that describes a group of acquired and inherited disorders of
keratinization, all of which are characterized by scaling. The most common form is
ichthyosis vulgaris, an autosomal dominant disorder that becomes apparent between 3
months and 5 years of age. Affected individuals develop thin scales that have elevated edges,
which create a pasted-on appearance, such as described for the boy in the vignette. The
extensor surfaces of the lower extremities are affected primarily, although scales may be
present on the trunk and upper extremities. Typically, the face and antecubital and popliteal
fossae are spared.
XX. Skin disorders, Self-Assessment Exercise by Category
PREP Archive on CD-ROM. Copyright 2003, American Academy of Pediatrics.

Dr

_F

aq

eh

Ichthyosis vulgaris is a chronic disorder that tends to improve over time. Although it
can be managed effectively, there is no cure, and scaling returns if therapy is discontinued.
Initially, treatment involves frequent use of emollients that prevent evaporation of moisture
from the skin. If the response to this therapy is not satisfactory, agents containing urea or
alpha hydroxy acids (eg, lactic acid, glycolic acid) may be applied to improve the skins
ability to bind water. Although these agents are effective, patients may experience burning or
stinging that could preclude their use. To control scale, preparations containing salicylic acid
may be prescribed.
As many as 50% of children who have ichthyosis vulgaris also have atopic dermatitis.
However, atopic dermatitis is a chronic, relapsing pruritic eruption composed of
erythematous papules or thin plaques, not simply scale. In young children who have atopic
dermatitis, the lesions are located in flexural areas (eg, the antecubital and popliteal fossae);
ichthyosis vulgaris primarily affects the extensor surfaces of the lower extremities.
Other forms of ichthyosis generally may be distinguished from ichthyosis vulgaris on
historical and clinical grounds. Bullous ichthyosis, also known as congenital bullous
ichthyosiform erythroderma or epidermolytic hyperkeratosis, is an autosomal dominant
condition that presents at birth with widespread scaling, redness, erosions, or bullae. After
infancy, the clinical appearance changes to include thick, yellow scales on the flexural
surfaces of elbows and knees as well as on the trunk, palms, and soles. Lamellar ichthyosis is
an autosomal recessive disorder that also is evident at birth. Newborns have a collodion
membrane, ectropion, and eclabium. Once the membrane is shed, the skin becomes
erythematous, and large, plate-like scales develop that involve the entire body.
Although psoriasis produces scale, its clinical appearance differs greatly from that of
ichthyosis vulgaris. Lesions of psoriasis are erythematous papules or plaques covered by a
thick, adherent scale. Typical sites of involvement include the scalp, ears, elbows, knees,
umbilicus, and gluteal cleft. The diaper area may be affected in infants, although scaling
often is absent. In older children or adolescents, pitting, yellowing, or thickening of the nails
may be observed.
References:
Rabinowitz LG, Esterly NB. Atopic dermatitis and ichthyosis vulgaris. Pediatr Rev.
1994;15:220-226
Shwayder T. Ichthyosis in a nutshell. Pediatr Rev. 1999; 20:5-12
Shwayder T, Ott F. All about ichthyosis. Pediatr Clin North Am. 1991;38:835-857
Critique 176

Preferred Response: B
[View Question]
An erythematous, tender, boggy mass on the scalp that is associated with alopecia, as
described for the boy in the vignette, almost always is a kerion, the inflammatory form of
tinea capitis. Tinea capitis is the most common cause of hair loss among children and
represents a significant health problem, particularly among African-Americans. In the United
States, 95% of these infections are caused by Trichophyton tonsurans; Microsporum canis is
responsible for most other cases. The initial stage of tinea capitis is not inflammatory and is
characterized by one or more patches of alopecia and scaling of the scalp or by diffuse
XX. Skin disorders, Self-Assessment Exercise by Category
PREP Archive on CD-ROM. Copyright 2003, American Academy of Pediatrics.

Dr

_F

aq

eh

scaling and less obvious hair loss. Several weeks later, an inflammatory stage may develop
that results in the formation of one or more kerions.
A kerion should be treated with griseofulvin in a dose of 15 to 20 mg/kg per day of
the microsize preparation administered orally once daily for 6 to 8 weeks. Lower doses of
griseofulvin may be ineffective. When inflammation is severe or extensive, some clinicians
advise the use of prednisone for 7 to 10 days. Complete hair regrowth occurs in the majority
of children who have inflammatory tinea capitis; rarely is there partial or complete alopecia
at the site of a healed lesion. Because lesions are tender and erythematous, exhibit pustules,
and often are associated with regional lymphadenopathy, they may be mistaken for a
bacterial abscess. However, the presence of alopecia and the absence of a history of trauma
make this diagnosis unlikely. Therefore, incision and drainage are not indicated. Although a
bacterial culture of the scalp often yields Staphylococcus aureus in children who have a
kerion, this finding is more likely due to colonization than to an actual infection. For this
reason, antibiotics active against S aureus, such as cephalexin, mupirocin, or oxacillin,
generally are not required.
References:
Frieden IJ. Diagnosis and management of tinea capitis. Pediatr Ann. 1987;16:39-48
Frieden IJ, Howard R. Tinea capitis: epidemiology, diagnosis, treatment, and control. J Am
Acad Dermatol. 1994;31:S42-S46
Pomeranz AJ, Fairley JA. Management errors leading to unnecessary hospitalization for
kerion. Pediatrics. 1994;93: 986-988
Stein DH. Tineas-superficial dermatophyte infections. Pediatr Rev. 1998;19:368-372
Critique 195

Preferred Response: B
[View Question]
The presence of erosions covered by a yellow or honey-colored crust suggests a diagnosis of
crusted (nonbullous) impetigo. In contrast, bullous impetigo is characterized by flaccid bullae
and round, erythematous, superficial erosions surrounded by a rim of scale, which is the
remnant of the roof of the bulla. The crusted form accounts for more than 70% of cases of
impetigo and is particularly prevalent in warm, humid climates.
Staphylococcus aureus is the sole pathogen recovered from 53% to 77% of patients
who have crusted impetigo. Both S aureus and group A beta-hemolytic Streptococcus
(GABHS) are recovered from 9% to 30% of children who have crusted impetigo; GABHS
alone is recovered from only 1% to 3%. Bullous impetigo always is due to infection by S
aureus, usually with strains belonging to phage group 2. These organisms produce an
epidermolytic toxin that damages intercellular adherence, causing a cleft high in the
epidermis and the formation of a fragile blister that ruptures rapidly.
When treating patients who have crusted impetigo, an antibiotic active against both S
aureus and GABHS should be selected. If the infection is widespread or multifocal, such as
described for the girl in the vignette, an oral agent, such as a first-generation cephalosporin
(eg, cephalexin) or dicloxacillin, is preferred. Erythromycin is an acceptable alternative
except in geographic areas where there is a high prevalence of resistant strains. Because
penicillin lacks activity against S aureus, it should not be selected for the treatment of either
XX. Skin disorders, Self-Assessment Exercise by Category
PREP Archive on CD-ROM. Copyright 2003, American Academy of Pediatrics.

Dr

_F

aq

eh

form of impetigo. Oral antimicrobial agents should be continued for 7 to 10 days. If the
infection is localized, a topical antibiotic such as mupirocin applied to affected areas three
times daily for 7 to 10 days could be used.
Two conditions that may be confused with crusted impetigo are herpes simplex virus
(HSV) infection and nummular eczema. Cutaneous HSV produces clustered vesicles on an
erythematous base. Although a crust forms as the vesicles rupture, the presence of grouped
individual vesicles or papules helps to differentiate this infection from impetigo. Most HSV
infections require only local care. In severe or widespread forms, an oral antiviral agent (eg,
acyclovir) may be employed; topical agents are less effective. Nummular eczema is
characterized by coin-shaped macules that may be crusted or occasionally oozing. Unlike
impetigo, lesions of nummular eczema are distributed symmetrically, persist for weeks to
months, and occur most commonly on the extremities. Treatment of nummular eczema
includes the application of a topical corticosteroid twice daily. Infants and young children
may be treated with a low-potency preparation (eg, hydrocortisone 1%); older children or
adolescents require a mid-potency agent (eg, triamcinolone 0.1%).
References:
Barton LL, Friedman AD. Impetigo: a reassessment of etiology and therapy. Pediatr
Dermatol. 1987;4:185-188
Darmstadt GL. A guide to superficial strep and staph skin infections. Contemporary
Pediatrics. 1997;14:95-116
Darmstadt GL. Oral antibiotic therapy for uncomplicated bacterial skin infections in children.
Pediatr Infect Dis J. 1997;16:227-240
Critique 218

Preferred Response: B
[View Question]
Staphylococcal scalded skin syndrome (SSSS) is a serious toxin-mediated consequence of
infection with certain strains of Staphylococcus aureus. These organisms, which usually
belong to phage group II, types 71 and 55, elaborate exfoliative toxins that spread
hematogenously from a focus of infection, often the nasopharynx or conjunctivae. Upon
reaching the skin, the toxin disrupts the granular cell layer, creating an intraepidermal blister.
As a consequence, the epidermis above may be lost, with the formation of erosions. A
spectrum of exfoliative toxin-induced disease exists. In the mildest form, bullous impetigo,
the toxin has a localized effect; in SSSS, the effects are widespread. An intermediate,
abortive form of SSSS, known as staphylococcal scarlet fever, also may occur.
SSSS often follows an upper respiratory tract infection and is characterized by the
sudden onset of fever, irritability, erythema, and tenderness of the skin, as described for the
infant in the vignette. The erythema is scarlatiniform, composed of fine papules that have a
sandpaper feel, and often begins in periorificial or flexural areas. Within 24 to 48 hours, the
skin begins to wrinkle and blisters. As the roofs of blisters are lost, erosions and crusting
appear. Crusting is particularly evident around the mouth. A useful clinical finding is
epidermal separation when rubbing apparently normal skin (the Nikolsky sign), as described
for the infant in the vignette. Injection of the conjunctivae and fissuring of the lips are
common. Patients who exhibit erythema without blistering or erosions are said to have
XX. Skin disorders, Self-Assessment Exercise by Category
PREP Archive on CD-ROM. Copyright 2003, American Academy of Pediatrics.

Dr

_F

aq

eh

staphylococcal scarlet fever.


The diagnosis of SSSS can be confirmed by performing a cutaneous biopsy or
examining a frozen section of a blister roof. Patients who have SSSS generally are treated
with an intravenous antibiotic active against S aureus (eg, oxacillin). Wound care, analogous
to that employed for burns, and careful attention to fluid and electrolyte balance also are
indicated. Recovery occurs in 5 to 7 days, and postinflammatory desquamation is common.
Due to the superficial nature of the blisters, scarring is unlikely.
Although a scald burn may mimic SSSS, the absence of a supportive history, the
presence of fever and conjunctival injection, the distribution of the eruption, and the presence
of the Nikolsky sign make this diagnosis unlikely. In the early erythrodermic phase, SSSS
may be confused with streptococcal scarlet fever. However, a sore throat, headache, palatal
petechiae, and pharyngeal exudate, which are characteristic of streptococcal scarlet fever, are
not seen in SSSS.
A major challenge is to differentiate SSSS from toxic epidermal necrolysis (TEN).
TEN is a potentially life-threatening reaction that usually is drug-induced and causes
full-thickness epidermal necrosis. The initial lesions of TEN are erythematous macules or
thin plaques that progress rapidly to form blisters, resulting in widespread epidermal loss.
When there is uncertainty about the diagnosis, a skin biopsy can distinguish between SSSS
and TEN.
Both staphylococcal and streptococcal toxic shock syndrome (TSS) produce fever and
macular erythema that may mimic SSSS. However, these conditions also cause hypotension,
myalgias, vomiting, diarrhea, and renal dysfunction. In addition, erosions and the Nikolsky
sign are not present in TSS.
References:
Manders SM. Toxin-mediated streptococcal and staphylococcal disease. J Am Acad
Dermatol. 1998;39: 383-400
Resnick SD. Staphylococcal toxin-mediated syndromes in childhood. Semin Dermatol.
1992;11:11-18
Scales JW, Fleischer AB Jr, Krowchuk DP. Bullous impetigo. Arch Pediatr Adolesc Med.
1997;151:1168-1169
Critique 237

Preferred Response: A
[View Question]
The presence of a persistent periorificial and acral eruption associated with diarrhea, sparse
hair, and failure to gain weight suggests the diagnosis of acrodermatitis enteropathica (AE).
AE is an autosomal recessive disorder in which a defective transport protein in the
gastrointestinal tract results in impaired absorption of dietary zinc. Because human milk
contains a protein that facilitates zinc absorption, clinical manifestations often appear after an
infant is weaned from human milk to formula or cow milk, as occurred with the infant
described in the vignette. Zinc deficiency also may occur in patients who have chronic
malabsorption due to cystic fibrosis or celiac disease or those who are maintained on
parenteral nutrition without zinc supplementation.
Zinc deficiency leads to altered keratin synthesis and the development of
XX. Skin disorders, Self-Assessment Exercise by Category
PREP Archive on CD-ROM. Copyright 2003, American Academy of Pediatrics.

Dr

_F

aq

eh

erythematous erosions or thin plaques in the diaper area; around the eyes, nose, and mouth;
or on the extensor surfaces of the knees and elbows. Crusting of the scalp and alopecia may
occur, and nails may be lost. Many infants exhibit diarrhea and poor weight gain.
The diagnosis of AE is confirmed by the finding of a low zinc level. With
supplementation (eg, zinc sulfate 5 mg/kg per day), cutaneous lesions improve rapidly. While
awaiting improvement, topical corticosteroid may be applied twice daily. Although AE tends
to improve with age, continued zinc supplementation often is required.
Atopic dermatitis in infants involves the face, trunk, and extremities, but in contrast to
AE, it may wax and wane in severity and typically spares the diaper area. Impetigo produces
crusted erosions, but rarely presents with the widespread, persistent disease that is
characteristic of AE. Both irritant and seborrheic dermatitis affect the diaper area. Unlike AE,
though, irritant dermatitis is limited to this area. Although the distribution of seborrheic
dermatitis and AE are similar, lesions of seborrheic dermatitis have a greasy scale and often
are present in flexural areas (eg, neck folds, retroauricular folds, axillae). Finally, unlike
infants who have AE, those who have any of these other dermatologic conditions appear
clinically well.
References:
Arnsmeier SL, Paller AS. Getting to the bottom of diaper dermatitis. Contemporary
Pediatrics. 1997;14:115-129
Hambridge M. Trace element deficiencies in childhood. In: Suskind RM, Lewinter-Suskind
L, eds. Textbook of Pediatric Nutrition. 2nd ed. New York, NY: Raven Press; 1993:115-126
Singalavanija S, Frieden IJ. Diaper dermatitis. Pediatr Rev. 1995;16:142-147
Slonim AE, Sadick N, Pugliese M, Meyers-Seifer CH. Clinical response to alopecia,
trichorrhexis nodosa, and dry, scaly skin to zinc supplementation. J Pediatr.
1992;121:890-895
Critique 260

Preferred Response: A
[View Question]
Acne is the most common skin disease of adolescence, affecting 85% or more of all
teenagers. Altered keratinization, androgens, bacteria, and abnormal sebum production result
in obstruction of pilosebaceous follicles and inflammation. The clinical correlates of these
pathologic processes include blackheads (open comedones) and whiteheads (closed
comedones), which are due to obstruction within follicles, and erythematous papules,
pustules, or nodules, which are due to inflammation. In some patients, scars or cysts may
form.
Deciding which medication(s) to prescribe is based on several factors, including the
types and numbers of lesions present and the severity and extent of the disease. In general,
the types of lesions (eg, inflammatory, obstructive, or both) dictate the class of medication
needed. For example, inflammatory lesions should be treated with an agent that possesses
antimicrobial activity (eg, benzoyl peroxide or a topical or oral antibiotic), while reducing the
number of comedones is accomplished best with the use of a medication that alleviates
obstruction within follicles (eg, benzoyl peroxide or a topical retinoid). Accordingly, for the
girl described in the vignette, who has mild mixed acne characterized by a few inflammatory
XX. Skin disorders, Self-Assessment Exercise by Category
PREP Archive on CD-ROM. Copyright 2003, American Academy of Pediatrics.

Dr

_F

aq

eh

lesions and comedones, the preferred initial treatment would be benzoyl peroxide 5% gel
topically. If only inflammatory papules and pustules are present, benzoyl peroxide can be
used if disease is mild; a topical (eg, clindamycin) or oral (eg, tetracycline) antibiotic can be
used if disease is moderate or severe. A patient who has only a few blackheads and
whiteheads may use benzoyl peroxide; more numerous obstructive lesions require therapy
with a topical retinoid (eg, tretinoin or adapalene). Salicylic acid wash may be used by
patients who have comedonal acne and cannot tolerate topical retinoids.
The severity of acne also guides therapy. Disease can be characterized as mild (a few
to several lesions), moderate (several to many lesions with a few nodules), or severe
(numerous lesions with many nodules). Although subjective and imprecise, this system
allows the clinician to develop an appropriate treatment plan. Patients who have mild
inflammatory acne, characterized by a few papules or pustules, usually respond to treatment
with benzoyl peroxide gel applied topically. In contrast, severe disease that involves large
numbers of inflammatory lesions or the presence of nodules or scarring often requires an oral
antibiotic.
Finally, the extent of lesions also influences therapeutic decisions. Modest numbers of
inflammatory lesions limited to the face may be treated with a topical antibiotic. However,
patients who have widespread involvement of the chest and back are treated best with an oral
antibiotic; it is impractical and costly to use topical antibiotics to treat large areas.
References:
Krowchuk DP. Treating acne: a practical guide for pediatricians. Adolescent Health Update.
1998;11:1-10
Leyden JJ. Therapy for acne vulgaris. N Engl J Med. 1997;336:1156-1162
Rothman KF, Lucky AW. Acne vulgaris. Adv Dermatol. 1993;8:347-375

1999 Self-Assessment Exercise


XX. Skin disorders
[Return to Category List]

Questions [Print Directions](3)


Question 4.

Answer.

By the year 2000, the lifetime risk of malignant melanoma in white Americans may exceed
1%. Although cases in childhood are rare, the incidence in adolescents appears to be
increasing.
Of the following, the cutaneous finding that MOST warrants an immediate skin biopsy to
exclude malignant changes is
A. a new round 2 mm brown mole on the back
B. a new round 4 mm red papule on the cheek
XX. Skin disorders, Self-Assessment Exercise by Category
PREP Archive on CD-ROM. Copyright 2003, American Academy of Pediatrics.

Dr

_F

aq

eh

C. an 8 mm mole with a firm nodule on one side


D. bleeding and crusting of a repeatedly traumatized 3 mm mole
E. development of a white halo around an existing mole
Question 27.

Answer.

You have been treating a 15-year-old girl for acne for more than 1 year. She has multiple
open and closed comedones, scattered 2 mm and 3 mm red papules, and rare pustules on her
face and upper back and chest. The acne has not responded to topical therapy, including
antibiotics, retinoids, and benzoyl peroxide.
Of the following, the best NEXT step in management is a trial of
A.
B.
C.
D.
E.

oral corticosteroids
oral isotretinoin
oral tetracycline
topical azelaic acid
topical salicylic acid

Question 53.

Answer.

A 14-year-old boy is brought to your office for evaluation of an itchy, scaly rash. You suspect
psoriasis and search for other typical findings
Of the following, the finding that is MOST suggestive of psoriasis is
A.
B.
C.
D.
E.

Darier sign
blistering of the palms and soles
hyperpigmented nails
Kobner or isomorphic phenomenon
periorificial erosions and crusts

Question 80.

Answer.

Ichthyosis vulgaris occurs in 3% to 5% of children. Coarse scales on the shins in the winter
is the most common manifestation of this autosomal dominant disorder. It can be associated
with widely disseminated scales on the trunk and extremities as well as keratosis pilaris on
the upper arms, thighs, and cheeks.
Of the following, the MOST effective treatment for the patients who have ichthyosis is
A.
B.
C.
D.
E.

benzoyl peroxide gel


hydrophilic ointment
lactic acid cream
tretinoin cream
triamcinolone cream

Question 106.
XX. Skin disorders, Self-Assessment Exercise by Category
PREP Archive on CD-ROM. Copyright 2003, American Academy of Pediatrics.

Answer.

Dr

_F

aq

eh

A 7-year-old girl develops widespread poison ivy characterized by linear red papules and
vesicles over her arms and legs. Seven days earlier she had been on a bicycle ride in the
woods with her dog. Her mother is concerned because the rash began 3 days ago and new
blisters are continuing to develop.
Of the following, the MOST likely explanation for the continued appearance of new lesions is
A.
B.
C.
D.
E.

development of a generalized eczematous reaction that is immune-mediated


exposure to aerosolized poison ivy allergen in the woods
ongoing reaction to poison ivy that she brushed against in the woods
re-exposure to poison ivy on the dogs hair
spread by exposure to fluid from ruptured blisters

Question 133.

Answer.

A 17-year-old boy develops an asymptomatic truncal rash consisting of 0.5 to 2.0 cm oval
scaly red patches with dusky centers. The rash follows the lines of cleavage and has been
increasing in intensity for 2 weeks. No new soaps have been introduced in the household, and
the boy is taking no medications. He has been sexually active with two partners.
Of the following, the most appropriate management is to
A.
B.
C.
D.
E.

begin a trial of moderate potency topical steroids on the most prominent lesions
counsel the patient about the self-limited rash and obtain a serologic test for syphilis
encourage sun exposure and consider phototherapy if the rash does not improve
encourage the use of lubricants, particularly following bathing and swimming
obtain a skin biopsy and delay recommendations until the results are available

Question 158.

Answer.

A healthy term infant is noted to have a port-wine stain that involves the first and second
branches of the trigeminal nerve. You suspect that the infant has Sturge-Weber syndrome.
Of the following, the BEST test to confirm this diagnosis is
A.
B.
C.
D.
E.

cerebrospinal fluid analysis


electroencephalography
head ultrasonography
magnetic resonance imaging scan
skull radiography

Question 183.

Answer.

At the annual health supervision visit of a 5-year-old girl, you note six caf au lait spots more
than 0.5 cm in diameter scattered on her trunk.
Of the following, the additional finding that best supports the diagnosis of
neurofibromatosis-1 is
A. a port-wine stain in the ophthalmic distribution of the trigeminal nerve
XX. Skin disorders, Self-Assessment Exercise by Category
PREP Archive on CD-ROM. Copyright 2003, American Academy of Pediatrics.

Dr

_F

aq

eh

B.
C.
D.
E.

Lisch nodules on the irides


multiple hemangiomas on the lower extremities
multiple hypopigmented macules on the trunk
swirls of marble-like hyperpigmentation on the back and chest

Question 238.

Answer.

A 14-year-old boy returns to your office for evaluation of a chronic foot dermatitis. During
the past year, he has had three episodes of cellulitis involving his feet. You also have noted a
rash on his hands on several occasions.
Of the following, the finding that is MOST suggestive of atopic dermatitis rather than tinea
pedis is
A. a chronic itchy symmetric dermatitis in both groin creases
B. flares that are associated with the development of papules and vesicles on the instep
of one foot
C. papules, vesicles, and lichenification symmetrically involving the palms and the soles
D. scaling and maceration in the fourth toe web spaces during symptom-free periods
E. symmetric pruritic rash restricted to the tops of both feet
Question 256.

Answer.

You prescribed penicillin for presumed streptococcal pharyngitis for a 10-year-old boy who
had fever and exudative tonsillitis. Following 5 days of therapy, he returns for evaluation of
a rash that began on his face and spread rapidly over his trunk and extremities. You suspect
Stevens-Johnson syndrome.
Of the following, the finding that MOST supports this diagnosis is
A.
B.
C.
D.
E.

a diffuse scarlatiniform rash


erythema of the palms, soles, conjunctivae, and oral mucosa
extensive blistering and erosions of the conjunctivae and oral mucosa
flaccid blisters on the abdomen and back
widespread erythema over the trunk and extremities

Answers
Critique 4

Preferred Response: C
[View Question]
When examining patients who have moles, a number of findings can be early warning signs
for malignant melanoma. They can be explained to children and parents as the ABCs of
moles. Asymmetry or irregularity in the shape of a new or preexisting mole should be noted.
Unusual changes in the Borders, such as the development of projections or invagina-tions,
may warrant a biopsy. Color changes, including darkening or lightening or the development
of various colors (blue, red, black, brown) in a nonuniform distribution within moles, and
XX. Skin disorders, Self-Assessment Exercise by Category
PREP Archive on CD-ROM. Copyright 2003, American Academy of Pediatrics.

Dr

_F

aq

eh

Diameter greater than 6 mm in an acquired mole require further evaluation. Itching and
burning in a mole that otherwise appears normal may be associated with inflammation in
early melanoma. In the absence of trauma, such lesions warrant continued evaluation and
possible biopsy. Accordingly, of the options given, an 8 mm mole with the projection of a
firm nodule on one side warrants immediate skin biopsy.
Other risk factors for melanoma include a family history of melanoma and
unusual-looking moles, blistering sunburns (particularly in early childhood), intermittent but
intense sun exposure, and geographic latitude of residence.
Malignant melanoma is the fastest growing malignancy in the United States, and a
good pediatric preventive health program should include counseling for parents and children
on these risk factors and the use of sun screen and other protective measures. A nevus with a
uniform white ring (halo nevus) is a benign nevus and does not warrant biopsy.
References:
Mehregan AH, Mehregan DA. Malignant melanoma in childhood. Cancer.
1993;71:4096-4103
Novakovic B, Clark WH Jr, Fears TR, Fraser MC, Tucker MA. Melanocytic nevi, dysplastic
nevi, and malignant melanoma in children from melanoma-prone families. J Am Acad
Dermatol. 1995;33:631-636
Orlow SJ. Melanomas in children. Pediatrics in Review. 1995;16:365-369
Precursors to malignant melanoma. National Institutes of Health Consensus Development
Conference Statement, Oct 24-26, 1983. J Am Acad Dermatol. 1984;10:683-688
Critique 27

Preferred Response: C
[View Question]
When a patient presents with severe acne, the clinician initially must obtain a detailed history
that includes menstrual history in females, drug history, family history, and review of any
topical overthecounter acne medications, makeup, moisturizers, and other topical
preparations the patient is using. A full examination to establish the extent and type of acne
lesions should be performed before devising a treatment plan.
Most adolescents and young adults who have comedonal and/or mild inflammatory
acne respond well to topical medications such as retinoids (eg, tretinoin, adapalene), benzoyl
peroxide, antibiotics (eg, clindamycin, tetracycline, erythromycin), and topical desquamating
agents (eg, salicylic acid). However, when patients who have inflammatory lesions, including
red papules and pustules, worsen or fail to improve when receiving topical preparations for
months, such as the girl described in the vignette, the clinician should consider adding oral
antibiotics. Tetracycline, tetracycline derivatives, and erythromycin are used most commonly
alone or in combination with topical medications.
Oral isotretinoin should be reserved for patients who have nodulocystic acne or
severe inflammatory acne that has failed treatment with topical medications, oral antibiotics,
and oral contraceptives. Patients receiving oral retinoids require close clinical and laboratory
monitoring, including a pregnancy test, to minimize the risk of serious adverse reactions.
Isotretinoin is a potent teratogen and is contraindicated during pregnancy. It should be
prescribed only for adolescent girls who also are using an effective form of contraception.
XX. Skin disorders, Self-Assessment Exercise by Category
PREP Archive on CD-ROM. Copyright 2003, American Academy of Pediatrics.

Dr

_F

aq

eh

Although topical and systemic steroids can trigger acne and should be avoided in most
patients, occasionally inflammation is so severe that oral corticosteroids are required for
several weeks at the beginning of a course of isotretinoin.
Azelaic acid is a new topical agent with keratolytic and antibiotic properties that can
be prescribed for patients who have comedonal and mild inflammatory acne. It can be used
alone or in combination with other topical medications as well as oral antibiotics.
References:
Hurwitz S. Acne vulgaris: pathogenesis and management. Pediatrics in Review.
1994;15:47-53
Leyden JJ. Therapy for acne vulgaris. N Engl J Med. 1997;336:1156-1162
Critique 53

Preferred Response: D
[View Question]
The Kbner or isomorphic phenomenon is characterized by primary skin lesions appearing in
areas of trauma. Although this reaction can be present in patients who have lichen planus,
vitiligo, and a number of other skin disorders, individuals who have psoriasis classically
demonstrate this phenomenon at some time during the evolution of their rash. The propensity
for psoriatic plaques to develop over the knees, elbows, and sacrum, which are areas of
trauma, probably is related to this reaction. Well-defined plaques also can erupt in surgical
scars, under casts, in burns, and in healing chickenpox lesions.
Other physical findings suggestive of psoriasis include: onycholysis (separation of the
nail plate from the underlying nail bed) and other nail dystrophy, thick scaly plaques in the
scalp, keratoderma (thickening of the palms and soles), Auspitz sign (capillary bleeding when
thick scale is pulled from silvery plaques on the skin), and resolution of skin lesions
following sun exposure. About 20% of those who have psoriasis report the onset of skin
disease before age 20, and many have a positive family history.
Darier sign is a diagnostic finding in patients who have mastocytosis. This finding can
be demonstrated by rubbing the lesions of solitary mastocytomas or the papules and nodules
of urticaria pigmentosa, which causes an overlying urticaria as a result of traumatic
degranulation of mast cells. In newborns and infants, edema can be intense enough to
produce blisters. Blistering of the palms and soles can occur in a number of conditions,
including acute contact dermatitis, tinea pedis, thermal and chemical injuries, and
epidermolysis bullosa. Hyperpigmentation of the nails can occur as a normal variant,
following the use of certain oral medications, and after exposure to a number of topical
preparations. Periorificial erosions and crusts are characteristic of several conditions,
including impetigo, syphilis, candidiasis, Stevens-Johnson syndrome, and nutritional
dermatoses such as acrodermatitis enteropathica (ie, zinc deficiency), essential fatty acid
deficiency, and biotin deficiency.
References:
Beylot C, Puissant A, Bioulac P, Saurat JH, Pinguet R, Doutre MS. Particular clinical
features of psoriasis in infants and children. Acta Derm Venereol Suppl (Stockh).
1979;87:95-97
XX. Skin disorders, Self-Assessment Exercise by Category
PREP Archive on CD-ROM. Copyright 2003, American Academy of Pediatrics.

Dr

_F

aq

eh

Koo J, Nguyen Q, Gambla C. Advances in psoriasis therapy. Adv Dermatol. 1997;12:47-72


Nyfors A, Lemholt K. Psoriasis in children. A short review and a study of 245 cases. Br J
Dermatol. 1975;92:437-442
Oranje AP, Marcoux D, Svensson A, et al. Topical calcipotriol in childhood psoriasis. J Am
Acad Dermatol. 1997;36:203-208
Sterns RS, Nichols KT. Therapy with orally administered methoxsalen and ultraviolet A
radiation during childhood increases the risk of basal cell carcinoma. A PUVA follow-up
study. J Pediatr. 1996;129:915-917
Critique 80

Preferred Response: C
[View Question]
Topical keratolytics induce desquamation of the skin and are designed to treat hyperkeratosis
(thickened skin), scaling, and xerosis. The safest, most effective agent for the treatment of
ichthyosis vulgaris is a keratolytic (eg, lactic acid cream).
Ichthyosis encompasses a number of disorders characterized by scaly, thickened skin
that are defined by specific inheritance patterns, clinical findings, course, and biochemical
and gene markers. Ichthyosis vulgaris, the most common variant, occurs in 3% to 5% of the
population and usually presents after 6 months of age with scaling of the distal extremities,
particularly the shins. Scaling can be diffuse, but the flexural creases of the arms and legs
usually are spared. This autosomal dominant disorder is associated with retention of scale at
the surface of the skin. Keratosis pilaris (scaly follicular papules on the upper arms, thighs,
and cheeks) often is present.
Keratolytics such as lactic acid, urea, glycolic acid, and other alpha-hydroxy acids
separate adherent scale from the underlying stratum corneum of the epidermis. They are
particularly effective in reducing the scale and pruritus associated with ichthyosis and
xerosis. Lubricants, including hydrophilic ointment and petrolatum, can be used as adjunctive
agents to help decrease water loss from the thickened but ineffective skin barrier associated
with ichthyosis.
Although benzoyl peroxide and tretinoin have keratolytic properties, they are
indicated primarily for patients who have acne vulgaris. Moreover, the high risk of irritation
and expense of these agents preclude their general use for treatment of ichthyosis.
Triamcinolone cream and other topical steroids are indicated for the treatment of
inflammatory dermatoses. Unless ichthyosis vulgaris is associated with atopic dermatitis,
topical steroids are of no value in the treatment of this usually noninflammatory condition.
References:
Rabinowitz LG, Esterly NB. Atopic dermatitis and ichthyosis vulgaris. Pediatrics in Review.
1994;15:220-226
Rand RE, Baden HP. The ichthyosesa review. J Am Acad Dermatol. 1983;8:285-305
Critique 106

Preferred Response: C
[View Question]

XX. Skin disorders, Self-Assessment Exercise by Category


PREP Archive on CD-ROM. Copyright 2003, American Academy of Pediatrics.

Dr

_F

aq

eh

Poison ivy exemplifies a type IV delayed hypersensitivity reaction that occurs in a patient
who has a contact allergic dermatitis. Following a period of sensitization of 1 to 2 weeks,
predisposed individuals develop a cutaneous reaction on re-exposure to the inciting allergen.
Itchy red macules and patches are followed by the appearance of papules, vesicles, and
bullae. Crusting and excoriations are typical, and edema, particularly on the face, hands, and
genitals, can become intense. Lesions tend to form linear and angulated patterns that
correspond to the sites of exposure. Areas covered by clothing usually are spared.
Following exposure to poison ivy, the skin rash can spread for several days.
Accordingly, the most likely explanation for the appearance of new lesions on the girl
described in the vignette is an ongoing reaction to poison ivy that she brushed against in the
woods. Areas of greatest exposure tend to develop the earliest and most intense rash. The
density of Langerhans cells, which process the contact allergen for the cell-mediated immune
system, normally varies over the skin surface. Factors that decrease the activity of these cells
will result in a delay or decrease in the allergic reaction. Moreover, Langerhans cells are not
present on mucous membranes, which are not involved in contact dermatitis. Poison ivy can
progress with chronic or re-exposure to the allergen from contaminated clothing or pet hair
and fur. Aerosolized exposure to poison ivy occurs when the plant is accidentally burned
with leaves in the fall. Highly sensitized individuals can develop a diffuse reaction on all
exposed areas, but there is no history of such an exposure in the girl in the vignette. Children
who have severe or widespread reactions rarely develop an immunologically mediated "id"
reaction in which a generalized rash appears even in areas that never were exposed to the
allergen. Blisters that appear in poison ivy exposure do not contain the allergen, and the fluid
from ruptured blisters cannot spread the rash.
As with other delayed hypersensitivity reactions, if poison ivy is not treated, the
reaction subsides over 2 to 4 weeks. Consequently, when serious reactions are treated,
therapy should be extended for at least 10 to 14 days. Mild localized reactions respond to
symptomatic treatment with cool tap water compresses, topical lotions, topical steroid
creams, and oral antihistamines. Severe localized or generally widespread reactions will
respond within 2 to 3 days of oral corticosteroid therapy. Steroid dosage should be tapered
slowly to reduce the risk of a rebound of skin rash and pruritus. Secondary infection can be
treated with topical or oral antibiotics, depending on the extent of involvement.
References:
Beltrani VS, Beltrani VP. Contact dermatitis. Ann Allergy Asthma Immunol.
1997;78:160-173
Hogan PA, Weston WL. Allergic contact dermatitis in children. Pediatrics in Review.
1993;14:240-243
Rudzki E, Rebandel P. Contact dermatitis in children. Contact Dermatitis. 1996;34:66-67
Critique 133

Preferred Response: B
[View Question]
The rash in the 17-year-old boy described in the vignette is most consistent with pityriasis
rosea (PR). Although this is a self-limiting dermatologic condition that does not require
therapy, a serologic test for syphilis should be obtained in any patient who might be sexually
XX. Skin disorders, Self-Assessment Exercise by Category
PREP Archive on CD-ROM. Copyright 2003, American Academy of Pediatrics.

Dr

_F

aq

eh

active to exclude the rash of secondary syphilis, which can masquerade as PR.
Although PR can occur at any age, children between ages 4 and 20 years account for
nearly 50% of the cases. Most cases are diagnosed in the winter and early spring, and some
are associated with mild flu-like prodromal symptoms.
PR typically begins with a single annular scaly patch that is 2 to 5 cm in size on the
trunk or proximal extremity, followed by a diffuse truncal eruption of round-to-oval scaly
patches that follow the lines of skin cleavage, resulting in a so-called "Christmas tree"
pattern. Although the distal extremities and face usually are spared, an atypical variant that
involves the face, hands, and feet (including the palms and soles) is more common in
African-American children.
In most children, PR resolves without therapy within 2 to 3 months. Some patients
may require administration of antihistamines, lubricants, or topical steroids to treat the
accompanying pruritus and scale. Sun exposure and artificial phototherapy also may suppress
inflammation and improve postinflammatory hyperpigmentation. However, treatment is not
necessary in most patients.
One third of patients who have syphilis develop secondary disease that is
characterized by generalized lymphadenopathy, flu-like symptoms, and a papulosquamous
eruption that typically involves the trunk, palms, soles, and mucous membranes. The rash of
secondary syphilis can last for several weeks to several months and, rarely, up to 1 year, and
many patients are completely asymptomatic.
Although histopathologic findings can help to distinguish PR and syphilis, the skin
biopsy findings are often nondiagnostic. Syphilis serology is a rapid, cost-effective screening
test for syphilis and should be considered in any sexually active individual who has a rash
consistent with PR.
References:
Allen RA, Janniger CK, Schwartz RA. Pityriasis rosea. Cutis. 1995;56:198-202
Cavanaugh RM Jr. Pityriasis rosea in children. Clin Pediatr. 1983;22:200-203
Dinulos JG, Graham EA. Influence of culture and pigment on skin conditions in children.
Pediatrics in Review. 1998;19:268-275
Critique 158

Preferred Response: D
[View Question]
Sturge-Weber syndrome is a sporadic neurocutaneous disorder. Manifestations of this
condition include venous leptomeningeal angiomatomas, seizures, and cognitive deterioration
that is variably progressive. Classically, there is a facial cutaneous nevus flammeus
(port-wine stain) in the distribution of the first and second branches of the trigeminal nerve
that covers at least one upper eyelid and the forehead. The port-wine stain is most frequently
unilateral, but may be bilateral. Magnetic resonance imaging (MRI) provides early diagnosis
of any leptomeningeal angiomatomas, and measurements of the choroid plexus provide
information about the extent of involvement in each hemisphere. However, the extent of
central nervous system (CNS) involvement does not always correlate with the size of the
cutaneous nevus flammeus. Of note, CNS involvement has been documented by MRI with
contrast enhancement even when no facial lesion was present. In an infant who has suspected
XX. Skin disorders, Self-Assessment Exercise by Category
PREP Archive on CD-ROM. Copyright 2003, American Academy of Pediatrics.

Dr

_F

aq

eh

Sturge-Weber syndrome, as described in the vignette, electroencephalography, cerebrospinal


fluid analysis, head ultrasonography, or skull radiography would not be as helpful as MRI to
confirm the diagnosis.
In patients who have Sturge-Weber syndrome, seizures occur in up to 90% of
patients. Most often, seizures are evident during the first year of life. Affected infants are at
high risk for cognitive sequelae; evidence of early developmental delay may be present in up
to 83% of infants. As the seizures worsen, there can be progressive cortical atrophy and
further cognitive decline. If left untreated, hemiplegia develops in up to 30% of the cases. In
appropriate candidates, partial hemispherectomy performed in infancy or early childhood
improves the outcome. Following surgery, children often can be tapered off of
anticonvulsants, cognitive improvement occurs, and there is little or no residual motor
weakness.
Ophthalmologic complications are common in patients who have Sturge-Weber
syndrome. Hemianopia can occur with or without hemiparesis. Of note, at least 30% of
children who have Sturge-Weber syndrome will develop glaucoma and 40% may have
evidence of a choroidal angioma. Buphthalmos (enlargement of the eye) also is common.
Thus, the expert care of an ophthalmologist who is familiar with this condition is essential.
References:
Gomez MR, Bebin EM. Sturge-Weber syndrome. In: Gomez MR, ed. Neurocutaneous
Diseases: A Practical Approach. Boston, Mass: Butterworths; 1987:356-367
Sujansky E, Conradi S. Sturge-Weber syndrome: age of onset of seizures and glaucoma and
the prognosis for affected children. J Child Neurol. 1995;10:49-58
Vining EP, Freeman JM, Pillas DJ, et al. Why would you remove half a brain? The outcome
of 58 children after hemispherectomy-the Johns Hopkins experience: 1968 to 1996.
Pediatrics. 1997;100:163-171
Critique 183

Preferred Response: B
[View Question]
Neurofibromatosis-1 (NF-1) is an inherited, autosomal dominant multisystem genodermatosis
with variable phenotypic expression. Clinical findings are evident in more than 30% of
individuals at birth and more than 50% by 2 years of age. Caf au lait spots (CALS), uniform
discrete tan-to-dark brown macules and patches, are the most common clinical sign of
disease. Six or more CALS at least 0.5 cm in diameter in children and 1.5 cm in adults is
suggestive of the diagnosis. In patients who have six or more CALS, the diagnosis of NF-1
can be confirmed by the presence of any of the following: Lisch nodules on the irides,
cutaneous and/or plexiform neurofibromas, axillary and/or inguinal freckling, optic glioma,
distinctive bony dysplasia, or a family history of neurofibromatosis in a first-degree relative.
Lisch nodules or iris hamartomas are present in about 50% of affected individuals by age 5
years and in all adults. Cutaneous neurofibromas begin to appear in adolescence and increase
in number and size through adult life. These soft, compressible tumors range in size from a
few millimeters to a few centimeters and are pushed easily into the surrounding skin,
demonstrating the "button hole" phenomenon. Axillary freckling consisting of multiple 2 to 4
mm brown macules is indistinguishable from small CALS and develops in about 20% of
XX. Skin disorders, Self-Assessment Exercise by Category
PREP Archive on CD-ROM. Copyright 2003, American Academy of Pediatrics.

Dr

_F

aq

eh

patients.
Although many patients who have NF-1 develop only cutaneous signs of disease,
life-threatening involvement of almost any organ system can occur. Moreover, there can be
wide variations in the expression of the disease even in individuals within the same family.
Affected children require careful follow-up, with special attention to neurodevelopmental
parameters, vision, hearing, and musculoskeletal findings. These patients also have an
increased risk of developing certain malignancies, such as neurofibrosarcoma, malignant
schwannoma, Wilms tumor, leukemia, and pheochromocytoma. All affected families should
receive genetic counseling.
A port-wine stain in the ophthalmic distribution of the trigeminal nerve is suggestive
of Sturge-Weber syndrome. Associated findings include ipsilateral leptomeningeal and
choroid vascular malformations, seizures, and glaucoma. Multiple hemangiomas on the lower
extremities can be associated with visceral hemangiomatosis. Although multiple
hypopigmented macules on the trunk are not a specific cutaneous finding for any condition, it
should prompt a thorough evaluation for tuberous sclerosis. Finally, swirling hyperpigmented
patches on the trunk can be a marker for the pigmented phase of incontinentia pigmenti. The
presence of other cutaneous findings should help to distinguish this disorder from
neurofibromatosis.
References:
Goldberg Y, Dibbern K, Klein J, Riccardi VM, Graham JM Jr. Neurofibromatosis type 1an
update and review for the primary pediatrician. Clin Pediatr. 1996;35:545-561
Johnson BL, Charneco DR. Cafe au lait in neurofibromatosis and in normal individuals. Arch
Dermatol. 1970;102:442-446
Riccardi VM. Neurofibromatosis and Albright's syndrome. In: Alper JC, ed. Genetic
Disorders of the Skin. St Louis, Mo: Mosby-Year Book, Inc; 1991:163-169
Critique 238

Preferred Response: C
[View Question]
A chronic foot dermatitis characterized by symmetric involvement of the palms and soles
with papules and lichenification is most suggestive of atopic dermatitis. Dyshidrotic eczema
is a term often used to describe a condition involving intensely pruritic, deep-seated papules
and vesicles on the palms, soles, sides of the feet, toes, and fingers of patients who have
atopy. Scratching results in formation of scale and lichenification. Severe dermatitis localized
to the hands and feet occurs more commonly in adolescents and adults, many of whom had
widespread eczema in infancy and early childhood. The development of impetigo and
cellulitis caused by Staphylococcus aureus is the most common complication of eczema in all
age groups and is the most likely explanation for the recurrent episodes of cellulitis described
in the vignette.
The groin creases generally are spared in patients who have atopic dermatitis. Irritant
or allergic contact dermatitis, tinea corporis, and candidiasis are common causes of dermatitis
involving the groin. Indeed, tinea cruris (ie, tinea of the groin creases) often is associated
with tinea pedis, which causes a foot dermatitis. However, the foot dermatitis in tinea pedis
tends to be asymmetric and spreads to the bottom and top of the feet from the web spaces of
XX. Skin disorders, Self-Assessment Exercise by Category
PREP Archive on CD-ROM. Copyright 2003, American Academy of Pediatrics.

Dr

_F

aq

eh

the toes. Subtle scaling and maceration restricted to the fourth toe web space during
symptom-free periods is typical of tinea pedis. Flares that are associated with the
development of papules and vesicles on the instep of one foot also are characteristic of tinea
pedis.
Allergic contact dermatitis causes a chronic pruritic dermatitis that usually involves
the tops of the feet symmetrically. Leather and chemicals used in shoe manufacturing are the
most common allergens, and avoidance of these products usually results in clearing of the
dermatitis. Occasionally, severe reactions spread over the entire foot and an "id reaction"
results in hand lesions, which can make clinical distinction from atopic dermatitis difficult. In
affected patients, patch testing may be required to make a specific diagnosis of allergic
contact dermatitis.
Juvenile palmar-plantar dermatosis or sweaty sock syndrome is characterized by
asymptomatic, symmetric, dry, scaly, fissured patches on the anterior plantar surfaces of the
feet. The balls of the feet and the bottoms of the great toes tend to be affected most severely.
Occasionally, similar lesions develop on the palms. Sweaty sock syndrome probably
represents an irritant dermatitis resulting from repeated hydration (when shoes are worn
during the day) and drying (when shoes are removed at night) of the thick stratum corneum
on the palms and soles.
References:
Oranje AP. Development of childhood eczema and its classification. Pediatr Allergy
Immunol. 1995;6(suppl 6):31-35
Rothe MJ, Grant-Kels JM. Atopic dermatitis: an update. J Am Acad Dermatol. 1996;35:1-13
Rosenthal JR. Pediatric fungal infections from head to toe: what's new? Curr Opin Pediatr.
1994;6:435-441
Critique 256

Preferred Response: C
[View Question]
Stevens-Johnson syndrome (SJS) often begins with a 1 to 2 day flu-like prodrome of fever,
sore throat, and general malaise followed by erythema, widespread blistering, and ulceration
of the skin and mucous membranes. Painful sloughing can involve the conjunctivae, oral and
nasal mucosae, urethra, vagina, and rectum. Rarely, patients develop lower airway disease
that can progress to respiratory distress syndrome. The rash tends to appear initially on the
head and neck as urticarial red macules and papules, which often develop central necrotic
blisters and ulcers. Cutaneous lesions may remain limited or spread over the face, neck,
trunk, and extremities.
Although no specific treatment is available for SJS, early diagnosis is necessary to
initiate supportive measures, discontinue medications or treat an infection that may have
triggered the process, and minimize the risk of life-threatening complications. Injury to the
skin and mucous membranes in SJS occurs at the dermal-epidermal junction and results in
full-thickness destruction of the epidermis and mucosa. Re-epithelialization of the skin and
mucous membranes can take several weeks and may be associated with scarring.
SJS must be distinguished from staphylococcal scalded skin syndrome (SSSS), which
presents with fever and tender generalized sunburn-like erythema followed by generalized
XX. Skin disorders, Self-Assessment Exercise by Category
PREP Archive on CD-ROM. Copyright 2003, American Academy of Pediatrics.

Dr

_F

aq

eh

flaccid bullae and erosions. Although the mucous membranes may be red, blistering and
sloughing do not occur. The cutaneous insult results from a soluble staphylococcal exotoxin
that binds to an antigen high in the epidermis just beneath the granular layer. As a
consequence, healing in uncomplicated cases is usually complete within 4 to 5 days. The
primary infection in SSSS usually involves superficial sites, such as the nares or
conjunctivae, and responds to oral antibiotics. However, in some cases, the infection may
originate from a deep soft-tissue, bony, or visceral infection and require the administration of
parenteral antibiotics.
A generalized scarlatiniform eruption also can appear in patients who have scarlet
fever, toxic shock syndrome, drug reactions, Kawasaki syndrome, and viral infections.
References:
Kakourou T, Klontza D, Soteropoulou F, Kattamis C. Corticosteroid treatment of erythema
multiforme major (Stevens-Johnson syndrome) in children. Eur J Pediatr. 1997;156:90-93
Noskin GA, Patterson R. Outpatient management of Stevens-Johnson syndrome: a report of
four cases and management strategy. Allergy Asthma Proc. 1997;18:29-32
Rauch D. The spectrum of erythema multiforme. Pediatrics in Review. 1996;17:63-64
Tay YK, Huff JC, Weston WL. Mycoplasma pneumoniae infection is associated with
Stevens-Johnson syndrome, not erythema multiforme. J Am Acad Dermatol.
1996;35:757-760

1998 Self-Assessment Exercise


XX. Skin disorders
[Return to Category List]

Questions [Print Directions](4)


Question 23.

Answer.

An 18-month-old child will not be allowed to eat or drink while undergoing an evaluation for
seizures. Findings on physical examination include weight, 13 kg; blood pressure, 96/60 mm
Hg; and pulse, 100 beats/min. Electrolyte levels are normal. You elect to administer
intravenous fluids to the child.
The maintenance sodium requirements of this child are BEST met by a solution containing
5% dextrose and a sodium concentration of
A.
B.
C.
D.
E.

0 mEq/L
30 mEq/L
60 mEq/L
90 mEq/L
120 mEq/L

XX. Skin disorders, Self-Assessment Exercise by Category


PREP Archive on CD-ROM. Copyright 2003, American Academy of Pediatrics.

Dr

_F

aq

eh

Question 57.

Answer.

You note multiple 3 mm yellow papules and pustules surrounded by an erythematous


urticarial base on the trunk of a 3-day-old girl. She is breastfeeding vigorously, and findings
on physical examination are otherwise normal.
A smear of the contents of a pustule MOST likely will show
A.
B.
C.
D.
E.

budding yeast
eosinophils
Gram-positive cocci in clusters
multinucleated giant cells
neutrophils

Question 97.

Answer.

An otherwise healthy 12-month-old boy has recurrent hives. Physical examination reveals
three hives on the lower back and several brown macules on the chest and abdomen. The
parents report that the hives recur in exactly the same location. You rub one of the brown
macules on the boy's chest and note that a hive develops.
Among the following, the MOST likely diagnosis is
A.
B.
C.
D.
E.

fixed drug eruption


incontinentia pigmenti
neurofibromatosis
nevus of Ota
urticaria pigmentosa

Question 141.

Answer.

At birth, an African-American boy has multiple pustules on his scalp, forehead, and
extremities and crusted lesions surrounded by brown rings on the lower back and buttocks.
All other findings on physical examination are normal.
Of the following, the MOST likely diagnosis is
A.
B.
C.
D.
E.

eosinophilic folliculitis
erythema toxicum neonatorum
varicella
staphylococcal pustulosis
transient neonatal pustular melanosis

Question 174.

Answer.

A 10-year-old African-American boy has had an itchy rash on the scalp for the past 6 weeks.
Physical examination reveals several scaly, crusted patches on the occiput; nontender
posterior cervical adenopathy; and a fine papular rash on the face and neck.
XX. Skin disorders, Self-Assessment Exercise by Category
PREP Archive on CD-ROM. Copyright 2003, American Academy of Pediatrics.

Dr

_F

aq

eh

The MOST appropriate management of this condition is to obtain a


A.
B.
C.
D.
E.

fungal culture and await culture results before starting therapy


fungal culture and start oral griseofulvin
fungal culture and start selenium sulfide shampoo
potassium hydroxide preparation and start clotrimazole cream
potassium hydroxide preparation and start ketoconazole shampoo

Question 209.

Answer.

A 13-month-old boy who was weaned from human milk 6 weeks ago has had a
vesiculobullous skin eruption for 1 month. The eruption is localized to the extremities and to
the areas around his eyes and anus. In addition, he has developed diarrhea and alopecia.
You diagnose acrodermatitis enteropathica.
When the parents ask about the cause of the disease, your BEST response is that it is
A.
B.
C.
D.
E.

a disorder of unknown etiology


an inherited defect of zinc metabolism
due to a bacterial infection
due to hypersensitivity to sunlight
the result of an allergy to cow milk

Question 252.

Answer.

A 16-month-old girl has had an itchy rash for 4 months. No one else in her home has a rash
or pruritus. Physical examination reveals extensive excoriations and widespread rash that
spares the diaper area, body creases, palms, and soles. You note several groups of 4 to 6 mm
edematous, round, red papules, some with central vesicles, that are organized in a linear
pattern.
The MOST likely cause of these findings is
A.
B.
C.
D.
E.

acropustulosis of infancy
bullous impetigo
flea bite reaction
herpes simplex virus infection
scabies infestation

Answers
Critique 23

Preferred Response: B
[View Question]
Fluid and electrolyte requirements are related directly to rate of metabolism. Changes in the
basal metabolic rate affect the rate of endogenous water production through the excretion of
urinary solutes, heat production, and oxidation of fats, proteins, and carbohydrates. Several
XX. Skin disorders, Self-Assessment Exercise by Category
PREP Archive on CD-ROM. Copyright 2003, American Academy of Pediatrics.

Dr

_F

aq

eh

systems are available for calculating fluid and electrolyte requirements in humans, but the
most widely used is the caloric method, as simplified by Holliday and Segar. This system
relates the expenditure of calories needed to prevent catabolism to body weight in a resting,
hospitalized patient as follows:
Fluid requirements are based on these caloric requirements at a rate of 100 mL/100
kcal expended. The requirements for both sodium and potassium are 2 to 4 mEq for each 100
kcal expended per day.
Using this method, the child in the vignette, who has no ongoing fluid or electrolyte
losses, should receive 1,150 mL of fluids containing 23 to 46 mEq each of sodium and
potassium. Accordingly, to provide 3 mEq sodium/100 kcal expended, a solution containing
30 mEq sodium per liter is required. Solutions without sodium are reserved for patients who
are anuric, where only insensible water losses are replaced.
References:
Adelman RD, Solhung MJ. Pathophysiology of body fluids and fluid therapy. In: Nelson WE,
Behrman RE, Kliegman RM, Arvin AM, eds. Nelson Textbook of Pediatrics. 15th ed.
Philadelphia, Pa: WB Saunders Co; 1996:185-222
Dabbagh S, Atiyeh B, Fleischmann LE, Gruskin AB. Fluid and electrolyte therapy in
children. In: Burg FD, Ingelfinger JR, Wald ER, Polin RA, eds. Gellis & Kagan's Current
Pediatric Therapy. 15th ed. Philadelphia, Pa: WB Saunders Co; 1996:424-434
Hellerstein S. Fluid and electrolytes: clinical aspects. Pediatrics in Review. 1993;14:103-115
Holliday MA, Segar WE. The maintenance need for water in parenteral fluid therapy.
Pediatrics. 1957;19:823-832
Critique 57

Preferred Response: B
[View Question]
Erythema toxicum is a benign pustular eruption occurring in more than one third of neonates
during the first week of life. Lesions can develop at up to 1 month of age, but most cases
occur within the first 10 days. Scattered yellow papules on an erythematous urticarial base
typically appear on the face, trunk, buttocks, and proximal extremities on the second or third
day of life, as noted for the infant in the vignette. New lesions appear from hour to hour, and
some papules become frankly pustular before drying within several days. Infants are
otherwise healthy and asymptomatic. Skin lesions are self-limiting and do not require
therapy. If the diagnosis is uncertain, laboratory studies, including skin biopsy and Tzanck
smear, using Wright or Wright-Giemsa stain, demonstrate sterile intraepidermal pustules
filled with eosinophils. Approximately 10% to 15% of affected infants also will develop a
transient blood eosinophilia.
Erythema toxicum can be distinguished from transient neonatal pustular melanosis,
whichoccurs primarily in African-American male newborns on the face, scalp, lower back,
buttocks, and extremities. In this disorder, drying lesions develop a collarette of scale that
desquamates, leaving postinflammatory hyperpigmented macules.
In miliaria, scattered vesicles and red papules develop on the skin of heat-stressed
newborns placed on warmers or bundled in occlusive clothing. In the most superficial
variant, which is characterized by shiny 2 to 3 mm vesicles, the lesions can be wiped away by
XX. Skin disorders, Self-Assessment Exercise by Category
PREP Archive on CD-ROM. Copyright 2003, American Academy of Pediatrics.

Dr

_F

aq

eh

the examining finger. When the sweat ducts deeper in the dermis are obstructed, red papules
or pustules occur in the skin creases and other occluded areas. Lesions disappear quickly
when the skin is cooled.
Incontinentia pigmenti presents in the first 2 weeks of life with urticarial papules and
vesicles in a linear pattern that conforms to the embryonic lines of Blaschko. Affected infants
often have eosinophils in the vesicles and blood eosinophilia. Over several weeks to several
months the inflammatory lesions evolve into warty papules and heal with postinflammatory
hyperpigmentation. Many of the affected children display extracutaneous involvement of the
eyes, nervous system, and dentition. Incontinentia pigmenti is inherited as an X-linked
dominant disorder and usually is lethal in males.
Benign pustulovesicular eruptions must be distinguished from bacterial and viral
infections that have potentially life-threatening complications. In staphylococcal pustulosis,
the pustules usually are localized to the diaper area or other sites of cutaneous inoculation.
Moreover, the pustules tend to be relatively large compared with the underlying area of
erythema, unlike the pustules in erythema toxicum. Gram stain reveals neutrophils and
Gram-positive cocci, and skin cultures grow Staphylococcus aureus. In severe cases, the
fragile bullae associated with sttaphylococcal bacteria rupture and coalesce. Infection with
other bacterial organisms, including group B Streptococcus, Pseudomonas aeruginosa,
Listeria monocytogenes, and other Gram-positive and Gram-negative pathogens, rarely
presents with embolic papules and pustules in the skin.
Cutaneous candidiasis typically begins with bright red papules and pustules in the
diaper area and other intertriginous areas. Potassium hydroxide scrapings of such lesions
show diagnostic pseudohypha and budding yeast. In disseminated fungal infection, lesions
can develop at any site.
In herpes simplex virus infection, clustered vesicles characteristically appear at the
end of the first week of life. Any cutaneous or mucous membrane site can be involved; the
primary lesions often develop at the presenting part of the infant at delivery or on any other
surface in which a minor injury causes a break in the skin (eg, scalp monitoring site or
forceps). Infants may appear well before the organism disseminates. Diagnosis can be made
at the bedside with a Tzanck smear, which demonstrates multinucleated giant cells. Viral
cultures, immunoperoxidase stains, and polymerase chain reaction studies are used to
confirm the diagnosis.
References:
Moisson YF, Wallach D. Pustular dermatoses in the neonatal period. Ann Pediatr.
1992;39:397-406
Plantin P, Delaire P, Gavanou J, Boog G, Guillet G. Benign cutaneous lesions observed
within the 1st 48 hours of life in 874 infants born at a maternity ward of a university hospital.
Apropos of 148 cases. Ann Dermatol Venereol. 1990;117:181-184
Critique 97

Preferred Response: E
[View Question]
Urticaria pigmentosa is a benign variant of mastocytosis that occurs most commonly in
childhood. Skin lesions appear at or shortly after birth in 50% of affected children, and
XX. Skin disorders, Self-Assessment Exercise by Category
PREP Archive on CD-ROM. Copyright 2003, American Academy of Pediatrics.

Dr

_F

aq

eh

almost all patients have developed some lesions by 1 year of age. About 50% of affected
individuals exhibit only a solitary mastocytoma; others develop a few to hundreds or, rarely,
thousands of lesions. Typical urticarial papules and plaques range in size from 0.5 cm to 2.0
cm and increase in prominence, number, or both during the first 2 years of life. Overlying
pigmentation may be subtle for the first several months of life, but most lesions become
identifiable by a well-defined tan patch with fuzzy edges.
Although a skin biopsy can be obtained to show the characteristic aggregation of mast
cells, diagnosis usually can be confirmed by the demonstration of a Darier sign in which an
urticarial plaque develops over one of the tan patches after stroking, as occurred with the
patient in the vignette. Other physical stimuli, such as cold or heat exposure or medications
(eg, aspirin, codeine), can trigger hive formation. In some children, particularly during
infancy, the edema can be intense enough to cause blistering. With increasing age, the
number of mast cells in cutaneous lesions tends to decrease, resulting in less frequent
urticaria formation. Lesions disappear in most children by late childhood or early adult life.
Unfortunately, some patients who have urticaria pigmentosa will develop severe
complications (eg, anaphylaxis, gastric ulcers, failure to thrive) from extensive cutaneous
involvement or visceral disease. Mediators, including histamine, heparin, and chemotactic
factors, rarely cause systemic symptoms that require treatment with antihistamines, antacids,
sodium cromolyn, or systemic steroids.
In a fixed drug eruption, recurrent erythema, edema, and occasionally blistering occur
at one or several fixed sites in the skin after challenge with a specific drug. Identical lesions
recur after re-exposure to the inciting drug. When the drug is withdrawn, the lesions heal,
leaving behind macules of intense postinflammatory hyperpigmentation. However, no Darier
sign is present. Similarly, the caf au lait spots typical of neurofibromatosis and the
bluish-green patches of nevus of Ota do not urticate with rubbing.
The distinctive pigmentation of incontinentia pigmenti that follows the lines of
Blaschko develops after several weeks to months of inflammatory urticarial papules, vesicles,
and crust formation. These lesions also are not altered by rubbing. Over several weeks to
months the inflammatory lesions evolve into warty papules and heal with postinflammatory
hyperpigmentation that is splashy and reticulated. Many of the affected children display
extracutaneous involvement of the eyes, nervous system, and dentition. Incontinentia
pigmenti is inherited as an X-linked dominant disorder and usually is lethal in males.
References:
Caplan RM. The natural course of urticaria pigmentosa. Analysis and follow-up of 112 cases.
Arch Derm. 1963;87:146-157
Guzzo C, Lavker R, Roberts LJ II, Fox K, Schechter N, Lazarus G. Urticaria pigmentosa.
Systemic evaluation and successful treatment with topical steroids. Arch Dermatol.
1991;127:191-196
Critique 141

Preferred Response: E
[View Question]
Transient neonatal pustular melanosis is a benign pustular eruption characterized by brown
macules, pustules, and vesicles that occurs in 1% to 4% of newborns, particularly in
XX. Skin disorders, Self-Assessment Exercise by Category
PREP Archive on CD-ROM. Copyright 2003, American Academy of Pediatrics.

Dr

_F

aq

eh

African-American males. Lesions usually are present at birth on the face, scalp, upper chest,
lower back, buttocks, and thighs, as in the infant in the vignette, although all areas of the
body can be involved. These small (2 to 4 mm) pustules and vesicles dry, leaving a collarette
of scale and postinflammatory hyperpigmentation that fades in several weeks to months. In
some infants, freckle-like macules are the only manifestation of the rash. In children who
have light pigmentation, the inflammatory lesions may heal with little or no
postinflammatory hyperpigmentation.
The diagnosis of transient neonatal pustular melanosis can be confirmed by Tzanck
smear (using either Wright or Wright-Giemsa stain), which shows sheets of neutrophils, rare
eosinophils, and no multinucleated giant cells. No bacterial organisms are seen on Gram
stain, and cultures are negative for bacteria and virus.
Other benign pustular eruptions can be excluded from the differential diagnosis by the
typical distribution and presence of postinflammatory pigmentary changes. In erythema
toxicum neonatorum, there is a predilection for involvement of the face, trunk, and proximal
extremities, and the pustules are filled with eosinophils. Eosinophils also are found in the
pustules of eosinophilic folliculitis, which usually involves hair follicles on the scalp and
forehead; the trunk and extremities also can be affected. Although eosinophilic folliculitis
occurs in older infants between 5 and 10 months of age, the condition is rare in neonates. The
eruption lasts for days to weeks and resolves without scarring. Unlike patients who have
erythema toxicum neonatorum and transient neonatal pustular melanosis, those who have
eosinophilic folliculitis tend to be irritable with signs of pruritus.
Newborns who have varicella usually appear ill, have a fever, and demonstrate the
characteristic widespread scattered vesicular rash. Tzanck smear reveals multinucleated giant
cells typical of viral infection. In staphylococcal pustulosis, 2 to 5 mm pustules are
surrounded by a small ring of erythema. Denuded lesions leave a wet base, and Gram stain
shows Gram-positive cocci and neutrophils. The rash usually starts in the diaper area or
around the umbilical cord. Herpes simplex virus (HSV) infection must be considered in
newborns who develop any vesicular or crusted lesions. However, the lesions tend to cluster
and may be associated with trauma (eg, site of scalp monitoring). Appropriate viral cultures
should be obtained if HSV infection is suspected.
References:
Merlob P, Metzker A, Reisner SH. Transient neonatal pustular melanosis. Am J Dis Child.
1982;136:521-522
Ramamurthy RS, Reveri M, Esterly NB, Fretzin DF, Pildes RS. Transient neonatal pustular
melanosis. J Pediatr. 1976;88:831-835
Critique 174

Preferred Response: B
[View Question]
The child described in the vignette very likely has tinea capitis. In the United States, scalp
fungal infection occurs most commonly in African-American school children. Endothrix
infection by Trichophyton tonsurans accounts for 95% of positive fungal cultures obtained
from the scalp. This organism invades the hair shaft, resulting in increased fragility and
breakage of the hair just above the skin surface. The areas of hair loss are studded with short
XX. Skin disorders, Self-Assessment Exercise by Category
PREP Archive on CD-ROM. Copyright 2003, American Academy of Pediatrics.

Dr

_F

aq

eh

hairs poking out of the hair follicles, giving the typical appearance of "black dot tinea".
Initially, scale formation and pruritus may be mild, but when the patient becomes sensitized
to the fungus, erythema, scale, and crusting increases. Lymphadenopathy and pruritus can
become severe. In highly sensitized individuals, a boggy, tender, inflamed plaque or kerion
appears. Many sensitized patients develop an itchy, papular, scaly eruption on the scalp, face,
neck, or extremities referred to as an "id" reaction. Occasionally this dermatitic reaction
spreads over the entire skin surface.
A 10-year-old African-American child who presents with this clinical picture should
have a culture for fungus obtained and then be started on oral griseofulvin empirically.
Fungal organisms can take 2 to 4 weeks to grow in culture, and therapy should not be delayed
while awaiting results. A potassium hydroxide preparation of scrapings from the involved
areas of the scalp also can help confirm the diagnosis, but this study is less reliable than
fungal culture.
Patients require 2 to 4 months of systemic therapy to eradicate tinea capitis.
Unfortunately, problems with compliance, poor gastrointestinal absorption of griseofulvin,
underdosing of the medication, and reexposure to the organism at home and school lead to
high rates of reinfection. Although application of shampoos containing 2.5% selenium sulfide
helps to decrease the time to negative cultures, it does not eradicate the infection. Similarly,
topical antifungal creams (eg, clotrimazole cream) are of little value in treating tinea capitis.
In children who have allergies to griseo-fulvin, ketoconazole has been approved as
alternative therapy, but it is not as effective as griseofulvin. Several new oral antifungal
medications, which have not yet been approved in children, may provide more effective
options than are currently available.
References:
Allen HB, Honig PJ, Leyden JJ, McGinley KJ. Selenium sulfide: adjunctive therapy for tinea
capitis. Pediatrics. 1982;69:81-83
Bronson DM, Desai DR, Barsky S, Foley SM. An epidemic of infection with Trichophyton
tonsurans revealed in a 20-year survey of fungal infections in Chicago. Am J Acad Dermatol.
1983;8:322-330
Rasmussen JE. Cutaneous fungus infections in children. Pediatrics in Review.
1992;13:152-156
Critique 209

Preferred Response: B
[View Question]
Acrodermatitis enteropathica is a multisystem disorder that results from an inherited defect of
zinc metabolism. Symptoms usually present in the first year of life, most often between 4 and
10 weeks of age in bottle-fed infants and after weaning in breastfed infants. The clinical
features include diarrhea, candidal skin infections, failure to thrive, and a dermatitis
characterized by vesiculobullous skin lesions. The skin lesions frequently are infected with
Candida, particularly perioral and perianal lesions. Alopecia, photophobia, conjunctivitis,
and blepharitis also are common. In patients who are not treated, the gastrointestinal disease
can result in malnutrition and death.
The zinc deficiency in this disorder is due to impaired zinc absorption that is related
XX. Skin disorders, Self-Assessment Exercise by Category
PREP Archive on CD-ROM. Copyright 2003, American Academy of Pediatrics.

Dr

_F

aq

eh

to decreased levels of picolinic acid, which is derived from tryptophan, in the gut.
Administration of zinc results in good outcomes. The inheritance pattern of this condition is
autosomal recessive, and parents of affected infants should be referred for genetic counseling
to discuss the recurrence risk in future pregnancies. There is no evidence that bacterial
infection, hypersensitivity to sunlight, or cow milk allergy plays any role in the development
of acrodermatitis enteropathica.
References:
Hurwitz S. Clinical Pediatric Dermatology: A Textbook of Skin Disorders of Childhood and
Adolescence. 2nd ed. Philadelphia, Pa: WB Saunders Co; 1993
Singalavanija S, Frieden IJ. Diaper dermatitis. Pediatrics in Review. 1995;16:142-147
Van Wouwe JP. Clinical and laboratory diagnosis of acrodermatitis enteropathica. Eur J
Pediatr. 1989;149:2-8
Critique 252

Preferred Response: C
[View Question]
The girl in the vignette, who presents with an itchy rash, has papular urticaria caused by flea
bites. The rash is comprised of edematous urticarial papules, and central crusting or
vesiculation may develop. Lesions can involve any body site, but covered areas generally are
spared. In infants, papular urticaria can involve the face, upper trunk, arms, and legs. In older
children, body areas closest to the ground, particularly the tops of the feet and the shins, are
the sites of the most lesions, which tend to form lines of three to four papules referred to as
"breakfast, lunch, and dinner."
Papular urticaria is a delayed hypersensitivity reaction to the insect bite. The lesions
typically progress from acute, inflammatory, itchy papulovesicular lesions to dry, scaly
papules and areas of postinflammatory hyper- and hypopigmentation over 2 to 4 weeks. Most
people develop a tolerance to the reaction after a period of years, which probably explains
why older members of an infested household who have likely been bitten do not develop a
similar rash.
Preventive measures are most effective in reducing the development of new lesions.
Pets should be treated and infested areas sprayed to reduce flea populations in the home. The
skin must be kept covered, and insect repellents can be used judiciously to protect the skin in
sensitized children. Topical steroids and antihistamines may provide some relief from acute
lesions, and secondary infection should be treated with appropriate topical or oral antibiotics.
The differential diagnosis in infants who have an itchy bullous eruption includes
bullous impetigo, herpes simplex virus infection, scabies, and acropustulosis of infancy. In
bullous impetigo, expanding superficial, crusted bullae and pustules usually develop in areas
of previous injury or rash, such as excoriated insect bites. Common areas of involvement
include the distal extremities, neck, face, skin creases, and periumbilical area. A Gram stain
of the contents of the blisters and pustules usually reveals neutrophils and Gram-positive
cocci; cultures frequently grow Staphylococcus aureus.
The cutaneous lesions of herpes simplex virus are clustered, uniform 2 to 3 mm
vesicles or punched-out erosions spreading from a site of primary inoculation. The eruption
evolves quickly over 1 to 2 days, and the child usually is febrile and miserable. Bacterial
XX. Skin disorders, Self-Assessment Exercise by Category
PREP Archive on CD-ROM. Copyright 2003, American Academy of Pediatrics.

Dr

_F

aq

eh

cultures obtained from vesicles are sterile and Gram stain shows no organisms.
Multinucleated giant cells are found on Tzanck smears.
Similar to papular urticaria, scabies and acropustulosis of infancy are both chronic
itchy rashes. In scabies, the rash is symmetric and widespread without sparing of covered
areas. On close observation, 3 to 4 mm linear papulovesicular burrows that contain the
female scabies mite can be identified. Burrows are identified readily in infants on the wrists,
palms, soles, trunk, and genital area.
Although acropustulosis also presents with a symmetric rash, the pruritic, round,
papulovesicular lesions are aggregated most densely on the hands and feet. The eruption is
episodic and usually disappears by age 2 to 3 years without therapy. If necessary,
high-potency topical steroids provide symptomatic relief.
References:
Heng MC, Kloss SG, Haberfelde GC. Pathogenesis of papular urticaria. J Am Acad
Dermatol. 1984;10:1030-1034
Honig PJ. Anthropod bites, stings, and infestations: their prevention and treatment. Pediatr
Dermatol. 1986;3:189-197

XX. Skin disorders, Self-Assessment Exercise by Category


PREP Archive on CD-ROM. Copyright 2003, American Academy of Pediatrics.

Dr

_F

aq

eh

Endnotes
1 (Popup - Popup)
To Print all questions and answers in this file, from the File Menu in the upper left-hand
corner of the screen, select Print, then select All, and click the OK button.
2 (Popup - Popup)
To Print all questions and answers in this file, from the File Menu in the upper left-hand
corner of the screen, select Print, then select All, and click the OK button.
3 (Popup - Popup)
To Print all questions and answers in this file, from the File Menu in the upper left-hand
corner of the screen, select Print, then select All, and click the OK button.
4 (Popup - Popup)
To Print all questions and answers in this file, from the File Menu in the upper left-hand
corner of the screen, select Print, then select All, and click the OK button.

XX. Skin disorders, Self-Assessment Exercise by Category


PREP Archive on CD-ROM. Copyright 2003, American Academy of Pediatrics.

Anda mungkin juga menyukai